Last visit was: 24 Apr 2024, 11:36 It is currently 24 Apr 2024, 11:36

Close
GMAT Club Daily Prep
Thank you for using the timer - this advanced tool can estimate your performance and suggest more practice questions. We have subscribed you to Daily Prep Questions via email.

Customized
for You

we will pick new questions that match your level based on your Timer History

Track
Your Progress

every week, we’ll send you an estimated GMAT score based on your performance

Practice
Pays

we will pick new questions that match your level based on your Timer History
Not interested in getting valuable practice questions and articles delivered to your email? No problem, unsubscribe here.
Close
Request Expert Reply
Confirm Cancel
SORT BY:
Date
Tags:
Show Tags
Hide Tags
Alum
Joined: 19 Mar 2012
Posts: 4341
Own Kudos [?]: 51447 [221]
Given Kudos: 2326
Location: United States (WA)
Concentration: Leadership, General Management
Schools: Ross '20 (M)
GMAT 1: 760 Q50 V42
GMAT 2: 740 Q49 V42 (Online)
GMAT 3: 760 Q50 V42 (Online)
GPA: 3.8
WE:Marketing (Non-Profit and Government)
Send PM
Most Helpful Reply
Alum
Joined: 19 Mar 2012
Posts: 4341
Own Kudos [?]: 51447 [87]
Given Kudos: 2326
Location: United States (WA)
Concentration: Leadership, General Management
Schools: Ross '20 (M)
GMAT 1: 760 Q50 V42
GMAT 2: 740 Q49 V42 (Online)
GMAT 3: 760 Q50 V42 (Online)
GPA: 3.8
WE:Marketing (Non-Profit and Government)
Send PM
Alum
Joined: 19 Mar 2012
Posts: 4341
Own Kudos [?]: 51447 [60]
Given Kudos: 2326
Location: United States (WA)
Concentration: Leadership, General Management
Schools: Ross '20 (M)
GMAT 1: 760 Q50 V42
GMAT 2: 740 Q49 V42 (Online)
GMAT 3: 760 Q50 V42 (Online)
GPA: 3.8
WE:Marketing (Non-Profit and Government)
Send PM
Alum
Joined: 19 Mar 2012
Posts: 4341
Own Kudos [?]: 51447 [54]
Given Kudos: 2326
Location: United States (WA)
Concentration: Leadership, General Management
Schools: Ross '20 (M)
GMAT 1: 760 Q50 V42
GMAT 2: 740 Q49 V42 (Online)
GMAT 3: 760 Q50 V42 (Online)
GPA: 3.8
WE:Marketing (Non-Profit and Government)
Send PM
Re: GMATCLUB VERBAL ATTACK [#permalink]
17
Kudos
37
Bookmarks
Expert Reply

"Assumption" Strategies



Which of the following is an assumption must for the argument to hold true?



Answer is a presupposition- a statement that is a must if the argument is true. Without that assumption, argument falls apart- it
is an unstated premise. You ABSOLUTELY NEED an assumption. If you don’t have an assumption, then the argument dies.

• Stimulus will always be an argument- since you can only strengthen a conclusion, not a fact set. Identify, isolate and assess
the premises and conclusion
• Focus on conclusion
• Information in stimulus is suspect. Reasoning errors are present which can be corrected by answer choice.
• Strong prephrases possible
• Answer choices can bring new information which is not present in stimulus.
Assumptions are NECESSARYfor the argument – author relies upon it, and is fully committed to it.
Assumptions are NEVERstated in the argument – if it is directly mentioned, it is not an assumption.
Assumption is a minimalist answer- if it has any information author is not committed to or sure of it will not be correct.
How Assumption Questions are different from Must be true questions? – Assumption is something that must be true BEFORE
the argument.


Incorrect Answer Choices:



1. No tie to the conclusion- uses words from conclusion and is broadly related, but is not an assumption necessary for the
argument. A sub set is ‘breaks up a category’- assumption talks of boys and girls vs. teachers, and answer choice talks of
boys vs. girls
Opposite answer- were this to be assumed, argument will be weakened! It is important for this not to be true for conclusion
to be correct!
Switching terms- shell game

2. Addresses premises only
Follows on from the conclusion, instead of conclusion following on from it.
Too broad – e.g. instead of cyclists, comment about athletes. This doesn’t HAVE to be true! An assumption has to be 100 %
true
Type of Assumptions: Supporter/Defender

3. Supporter: Traditional linking role- links two part of premises, or premise to conclusion. Conclusion in such an argument
will have some new element which is not present in premises. In an assumption question, always be on lookout for a gap
in the argument premise and conclusion- a new element, and then the answer choice that brings that new element in
picture would be a straight win.

4. Defender: protect the argument by eliminating ideas that could weaken the argument. In order to believe that his
argument is correct and fool-proof, the author is assuming that all possible objections have been considered and rejected.
This can be an infinite range of assumptions.
If confused between some answer choices, use ASSUMPTION NEGATION TECHNIQUE

5. Converts an assumption question to a weaken question- negate both the assumption answer choices, the one whose negation causes
the entire argument to fall is the correct answer choice. This will happen because of necessary, indispensable nature of assumption.


Negation:

Is logical negation, not polar opposite. ‘I went to beach every day’ negated is ‘I didn’t go to beach every day’, not ‘I didn’t go to
beach any day’ (polar opposite)

Logical opposite of sweet is not sweet (not sour). Logical opposite divides the subject under consideration into two sets which
include all possible states.

Logical opposite of all – not all (not none); logical opposite of none is some. If all is 100, not all is 0-99. None is 0, and some is 1-100

Logical opposite of none- at least one.

You can use not necessarily for negating in most cases.


Some patterns observed in GMAT:

• At least one or at least some: When an assumption answer choice starts with these, it is normally correct. Negate and test –
none.
Constructions which claim to be most important/primary consideration/ main factor are normally wrong.
Look out for constructions with not/negative etc. they might be correct.

Assumptions and Causality



• Author assumes no other cause exists
• Author assumes cause always leads to effect
• Author assumes that when cause doesn’t occur effect doesn’t occur
• Eliminates the possibility that stated relationship is reverse
• Eliminates possible problems with data


In a cause- effect relationship argument, choose an assumption which supports this causal relationship in one of the above
5 ways. Answer choices discussing matters before or after the causal relationships are not correct.

In a fill in the bank question stem beginning with a premise indicator like ‘because_______ or as a result of _____’ you have to
provide an assumption. Beginning with conclusion indicator- ‘therefore ______’ you have to provide a must be true/main point
answer.

GMATPREP EXAMPLES



Q. If a person chooses to walk rather than drive, there is one less vehicle emitting pollution into the air than there would be
otherwise. Therefore if people would walk whenever it is feasible for them to do so, the pollution will be greatly reduced. Which
of the following is an assumption on which the argument depends?

A) Cutting down on pollution can be achieved in a variety of ways.
B) Taking public transportation rather than driving is not always feasible
C) Walking is the only feasible alternative to driving that results in reduction in pollution
D) There are people who never drive but who often walk
E) People sometimes drive when it is feasible to walk instead
Remember that an assumption is a statement that you absolutely need to be true.
C says there is no other way to reduce pollution. Argument doesn’t assume that. It is not a necessary statement, the
passage just says that walking will reduce pollution, not that only walking will reduce pollution.
E says that people sometimes currently drive when it is feasible to walk. only if this is true can this be stopped, and thus
pollution will be reduced.
Notice this is a pretty bland statement – you would think you already knew this! But actually you also assumed it
unconsciously. Thus assumptions for the most part would be something even you would have taken for granted.
Assumption statements will be weaker/ more moderate statements. in Strengthen/ Weaken more extreme statements are
good.
Using assumption negation technique: c reversed says that there are other ways, that doesn’t affect the argument. E
negated says that there is nobody who drives when it is feasible to walk i.e. everybody who can walk is already walking, then
the argument fails – the plan will have no effect.


Q. A famous singer recently won a lawsuit against an advertising firm for using another singer in a commercial to evoke the
famous singer’s well-known rendition of a certain song. As a result of the lawsuit, advertising firms will stop using imitators
in commercials. Therefore, advertising costs will rise, since famous singers’ services cost more than those of their imitators.
The conclusion above is based on which of the following assumptions?

(A) Most people are unable to distinguish a famous singer’s rendition of a song from a good imitator’s rendition of the same
song.
(B) Commercials using famous singers are usually more effective than commercials using imitators of famous singers.
(C) The original versions of some well-known songs are unavailable for use in commercials.
(D) Advertising firms will continue to use imitators to mimic the physical mannerisms of famous singers.
(E) The advertising industry will use well-known renditions of songs in commercials.
Reversing B says that renditions with commercial singers are no more effective. This has no effect on the costs of the
advertising which the argument is concerned with. This is out of scope. An assumption has to be within the scope of the
argument.
C reversed says that all original versions are available – but these are done by famous singers. So they will have to be paid.
E reversed says that ads will not use renditions of famous songs. If songs themselves are not used, they cannot lead to
increased advertising costs.
Answer: E


"ASSUMPTION" SET




1. To decrease the number of crimes in city Y, the city’s Police Commissioner proposed taking some police officers
from low-crime districts of the city and moving them to high-crime districts of the city. His proposal is based on city
Y crime data that show that the number of crimes in any district of the city decreases when additional police
officers are moved into that district. The Police Commissioner’s proposal depends on which of the following
assumptions?

• City X experienced a drastic reduction in crime after implementing a proposal similar to that proposed by the Police
Commissioner of city Y.
• The severity of crimes committed in any district of the city decreases when additional police officers are moved
into that district.
• The number of crimes committed in all high-crime districts of city Y is more than triple the number of crimes
committed in all low-crime districts of city Y.
• There are more low-crime districts than high-crime districts in city Y.
• Districts of the city from which police officers are removed do not experience significant crime increases shortly
after the removal of those officers.

2. Although there has been great scientific debate for decades over global warming, most scientists now agree that
human activity is causing the Earth’s temperature to rise. Though predictions vary, many global warming experts
believe that average global temperatures will rise between three and eight degrees Fahrenheit during the next
century. Such an increase would cause an alarming rise in sea levels, displacing millions of people by destroying
major population centers along the world’s coastlines. Which of the following is an assumption in support of the
argument’s conclusion?

• New technological developments in the next century will not divert rising seas from the world’s coastal cities.
• Individuals will not become more aware of the steps they can take to reduce the emission of greenhouse gases.
• Rising sea levels similarly affect all coastal population centers.
• Some global warming experts predict a greater than eight degree Fahrenheit increase in global temperatures
during the next century.
• Human activity is the sole cause of increasing global temperatures.

3. “The new HitItFar driver is the only golf club that uses space-age Titanium-Kryptonium alloy to strengthen its head.
This driver is SO good, the last 12 winners of the major tour championships have all recently switched to it! Isn’t
it time for you to add power to your swing and distance to your drives? Trade in your old driver today — and
HitItFar tomorrow!” All of the following claims are either implied or made explicitly in the above advertisement
EXCEPT:

• Switching to the HitItFar driver will improve your play.
• The HitItFar driver helped the last 12 major championship winners achieve their victory.
• Major championship winners are experts and know what constitutes a great golf club.
• Your existing driver is inferior to the HitItFar driver.
• Only HitItFar golf clubs have Titanium-Kryptonium alloy in the head of their drivers.

4. Researchers studying the spread of the Black Plague in sixteenth-century England claim that certain people survived
the epidemic because they carried a genetic mutation, known as Delta-32, that is known to prevent the bacteria
that causes the Plague from overtaking the immune system. To support this hypothesis, the researchers tested the
direct descendants of the residents of an English town where an unusually large proportion of people survived the
Plague. More than half of these descendants tested positive for the mutation Delta-32, a figure nearly three times
higher than that found in other locations. The researchers’ hypothesis is based on which of the following assumptions?

• Delta-32 does not prevent a carrier from contracting any disease other than the Plague.
• The Plague is not similar to other diseases caused by bacteria.
• Delta-32 did not exist in its current form until the sixteenth century.
• No one who tested positive for Delta-32 has ever contracted a disease caused by bacteria.
• The Plague does not cause genetic mutations such as Delta-32.

5. The popular notion that a tree’s age can be determined by counting the number of internal rings in its trunk is
generally true. However, to help regulate the internal temperature of the tree, the outermost layers of wood of the
Brazilian ash often peel away when the temperature exceeds 95 degrees Fahrenheit, leaving the tree with fewer
rings than it would otherwise have. So only if the temperature in the Brazilian ash’s environment never exceeds 95
degrees Fahrenheit will its rings be a reliable measure of the tree’s age. Which of the following is an assumption
on which the argument above depends?
• The growth of new rings in a tree is not a function of levels of precipitation.
• Only the Brazilian ash loses rings because of excessive heat.
• Only one day of temperatures above 95 degrees Fahrenheit is needed to cause the Brazilian ash to lose a ring.
• The internal rings of all trees are of uniform thickness.
• The number of rings that will be lost when the temperature exceeds 95 degrees Fahrenheit is not predictable.

6. The Department of Homeland Security has proposed new federal requirements for driver’s licenses that would
allow them to be used as part of a national identification system. Using licenses for purposes not directly related
to operating a motor vehicle is un-American because it would require U.S. citizens to carry the equivalent of
“papers.” Such a requirement would allow the government to restrict their movements and activities in the manner
of totalitarian regimes. In time, this could make other limits on freedom acceptable. The author assumes which of
the following?

• The next presidential election will be dishonest, as has happened in eastern European countries.
• The government will soon start curtailing the activities of those it considers “dissidents.”
• Blanket restrictions on law-abiding individuals are contrary to the traditions of American culture and law.
• The majority of Americans are not willing to give up their right to travel and move about without identification.
• Americans should resist all government regulation of their lives.12

7. Since the new publisher took control, a news magazine’s covers have featured only models and movie stars.
Previously, the covers had displayed only politicians, soldiers, and business leaders. A leading gossip columnist
claimed that the changes made the magazine relevant again. However, many newspaper editorials disagreed and
suggested that the new publisher is more interested in boosting sales than in reporting important news events.
Which of the following is an assumption necessary for the argument made by the gossip columnist’s opponents?

• The charitable activities of models and movie stars often focus public attention on pressing problems.
• Final authority for choosing the cover subject of the magazine lies with the publisher.
• A magazine can boost sales while highlighting the coverage of important world leaders.
• Some of the movie stars featured are now running for political office.
• Magazine issues with models or movie stars on the covers are purchased at a rate more than three times greater
than is the case with issues featuring politicians on the covers.

8. In response to the increasing cost of producing energy through traditional means, such as combustion, many utility
companies have begun investing in renewable energy sources, chiefly wind and solar power, hoping someday to
rely on them completely and thus lower energy costs. The utility companies claim that although these sources
require significant initial capital investment, they will provide stable energy supplies at low cost. As a result, these
sources will be less risky for the utilities than nonrenewable sources, such as gas, oil, and coal, whose prices can
fluctuate dramatically according to availability. The claim of the utility companies presupposes which of the following?

• The public will embrace the development of wind and solar power.
• No new deposits of gas, oil, and coal will be discovered in the near future.
• Weather patterns are consistent and predictable.
• The necessary technology for conversion to wind and solar power is not more expensive than the technology
needed to create energy through combustion.
• Obtaining energy from nonrenewable sources, such as gas, oil and coal, cannot be made less risky.

9. Surveys consistently show that the best-selling ice cream flavor is vanilla, although those who prefer chocolate
rarely order vanilla. Vanilla-flavored candy, then, probably sells better than chocolate-flavored candy. Which of
the following is an assumption upon which the author of the argument relies?

• Because someone prefers vanilla ice cream does not mean he prefers vanilla-flavored candy.
• Children who prefer vanilla ice cream also tend to like chocolate candy.
• Those who prefer neither vanilla nor chocolate ice cream also prefer other flavors of candy.
• Someone who prefers vanilla ice cream may still order chocolate on occasion.
• Preferences for certain ice cream flavors are similar to preferences for candy flavors.

10. The media claim that the economy is entering a phase of growth and prosperity. They point to lower unemployment
rates and increased productivity. This analysis is false, though. The number of people filing for bankruptcy has
increased every month for the last six months, and bankruptcy lawyers report that they are busier than they have
been in years. Which of the following is an assumption on which the argument depends?

• Unemployment rates are not useful indicators of growth and prosperity.
• Economic growth cannot be measured in terms of productivity.
• Legislation has not been recently passed to make legal bankruptcy easier to obtain.
• There has not been an increase in the number of bankruptcy lawyers..
• The media often misrepresent the current state of economic affairs.
Alum
Joined: 19 Mar 2012
Posts: 4341
Own Kudos [?]: 51447 [44]
Given Kudos: 2326
Location: United States (WA)
Concentration: Leadership, General Management
Schools: Ross '20 (M)
GMAT 1: 760 Q50 V42
GMAT 2: 740 Q49 V42 (Online)
GMAT 3: 760 Q50 V42 (Online)
GPA: 3.8
WE:Marketing (Non-Profit and Government)
Send PM
Re: GMATCLUB VERBAL ATTACK [#permalink]
8
Kudos
36
Bookmarks
Expert Reply
i dont have any set as such!
but I can post a couple of random questions on the topic

In the United States, of the people who moved from one state to another when they retired, the percentage who retired to Florida has decreased by three percentage points over the past ten years. Since many local businesses in Florida cater to retirees, this decline is likely to have a noticeably negative economic effect on these businesses.
Which of the following, if true, most seriously weakens the argument?
A. Florida attracts more people who move from one state to another when they retire than does any other state.
B. The number of people who move out of Florida to accept employment in other states has increased over the past ten years.
C. There are far more local businesses in Florida that cater to tourists than there are local businesses that cater to retirees.
D. The total number of people who retired and moved to another state for their retirement has increased significantly over the past ten years.
E. The number of people who left Florida when they retired to live in another state was greater last year than it was ten years ago.
User avatar
Manager
Manager
Joined: 02 Jun 2009
Status:Fighting again to Kill the GMAT devil
Posts: 80
Own Kudos [?]: 162 [26]
Given Kudos: 48
Location: New Delhi
Concentration: MBA - Strategy, Operations & General Management
 Q44  V28 GMAT 2: 650  Q49  V29 GMAT 3: 650  Q47  V33
WE 1: Oil and Gas - Engineering & Construction
Send PM
Re: WEEK 1: CRITICAL REASONING STRENGTHEN/WEAKEN Questions [#permalink]
1
Kudos
25
Bookmarks
souvik101990 wrote:
[textarea]

WEEK 1: Strengthen/Weaken/Evaluate the Argument


STRATEGY : Strengthen Question




In the Strengthen Set I got 4 Wrongs - #1,3,7 and 8 out of 9, I had seen 1 question earlier so not counting it.

Took 21 Minutes to did these 9 questions.


In # 1 and # 7 I was totally bowled over, could not get understand the argument properly.

I feel #3 was a sitter, I could not have missed that.

Souvik, any suggestions on how to improve on Strengthen questions.
Alum
Joined: 19 Mar 2012
Posts: 4341
Own Kudos [?]: 51447 [20]
Given Kudos: 2326
Location: United States (WA)
Concentration: Leadership, General Management
Schools: Ross '20 (M)
GMAT 1: 760 Q50 V42
GMAT 2: 740 Q49 V42 (Online)
GMAT 3: 760 Q50 V42 (Online)
GPA: 3.8
WE:Marketing (Non-Profit and Government)
Send PM
Re: GMATCLUB VERBAL ATTACK [#permalink]
4
Kudos
16
Bookmarks
Expert Reply

Meaning/clarity



1. American Heart Association researchers have calculated that one person in the United States
should experience a coronary event every 26 seconds.


• one person in the United States should experience a coronary event every 26 seconds
• a person in the United States should experience a coronary event once in every 26 seconds
• a coronary event will strike one person in the United States once in every 26 seconds
• every 26 seconds a person in the United States will experience a coronary event
• every 26 seconds a person in the United States should experience a coronary event
1. The original sentence says something that differs from the logical intent. The verb
“should” implies obligation; in this sentence, it indicates that one person in the United
States ought to experience a coronary event every 26 seconds, as though the person
deserves it, or, for that matter, as though any one person could continue indefinitely
to have such frequent heart attacks. The American Heart Association clearly means
that some person in the United States will experience a coronary event roughly every
26 seconds.
(A) This choice is incorrect as it repeats the original sentence.
(B) “Should” suggests that a person oughtto experience a coronary event, rather than
that a person will. Furthermore, “every 26 seconds” is an approximation, but the
phrase “once in every 26 seconds” is too precise for the situation, suggesting
coronary events occur with predetermined frequency.
(C) “Every 26 seconds” is an approximation, but the phrase “one person in the United
States once in every 26 seconds” is too precise for the situation, suggesting that a
specific person will suffer coronary events on a predetermined schedule.
Furthermore, "once in" is wordy and unnecessary.
(D) CORRECT. In this sentence, “will experience a coronary event” is free of the
unintended connotations of “should experience a coronary event.”
(E) “Should” suggests that a person ought to experience a coronary event, rather than
that a person will.

2. By choosing glass apartments towering a hundred feet over brownstone units designed for
earlier generations, seemingly younger-than-ever moneyed professionals have embraced a
modern design ethic that accentuated
their luxury-laden lives.

• By choosing glass apartments towering a hundred feetover brownstone units designed for
earlier generations, seemingly younger-than-ever moneyed professionals have embraced a
modern design ethic that accentuated
• By choosing glass apartments towering a hundred feet over brownstone units designed for
earlier generations, seeming younger-than-ever moneyed professionals have embraced a
modern design ethic that accentuates
• In choosing glass apartments in hundred-foot towersinstead of brownstone units designed for
earlier generations, seemingly younger-than-ever moneyed professionals have embraced a
modern design ethic that accentuates
• In choosing glass apartments in hundred-foot towersinstead of brownstone units designed for
earlier generations, seemingly younger-than-ever moneyed professionals have embraced a
modern design ethic that accentuated
• In choosing glass apartments towering a hundred feetover brownstone units designed for
earlier generations, seeming younger-than-ever moneyed professionals have embraced a
modern design ethic, accentuating
The original sentence contains two problems. First,“towering a hundred feet
over brownstone units” is unclear and implies the absurd meaning that the glass
apartments are located directly over brownstone units in different buildings. Second,
the verb "accentuated" should be in the present tense, since the earlier use of the
present perfect tense ("have embraced") implies that the embracing is still happening,
and therefore that the ethic accentuates the lives in the general present. Incidentally,
in this context, "by choosing" and "in choosing" have nearly identical meanings; as a
result, this split is immaterial.
(A) Incorrect, as it repeats the original sentence.
•(B) This choice repeats the errors from the choice(A) and adds another. The
adjective “seeming” is incorrect, since adjectives modify nouns; it is not
“seeming professionals,” but “seemingly younger... professionals.” An adverb
must be used to describe an adjective.
•(C) CORRECT. Using “in hundred-foot towers instead of” rather than “towering
a hundred feet over” makes the intended meaning clearer. Also, the verb
"accentuates" is in the proper tense (present).
•(D) The verb "accentuated" should not be in the past tense, as noted above.
•(E) In this choice, “towering a hundred feet over brownstone units” is unclear
and implies the absurd meaning that the glass apartments are located directly
over brownstone units in different buildings. The adjective “seeming” is
incorrect, as noted earlier in choice B. An adverb must be used to describe an
adjective. Finally, the participle "accentuating" should arguably be
replaced with the relative clause "that accentuates"; following a comma,
the participle implies that the professionals are doing the accentuating,
rather than the design ethic. This change of meaning is inadvisable.

3. Nearly 2000 years after its initial construction, the United Nations declared the Roman
aqueduct of Segovia to be a Heritage of Humanity in1985, prompting the Spanish government
to begin renovations on the aqueduct, which had been deteriorating.


• Nearly 2000 years after its initial construction, the United Nations declared the Roman
aqueduct of Segovia to be a Heritage of Humanity in1985, prompting the Spanish government
to begin renovations on the aqueduct, which had been deteriorating.
• Since its initial construction nearly 2000 years earlier, the Roman aqueduct of Segovia had
been deteriorating, prompting the Spanish government to begin renovations after the United
Nations declared the aqueduct to be a Heritage of Humanity in 1985.
• After being declared a Heritage of Humanity by the United Nations in 1985, the Spanish
government began renovations on the Roman aqueduct of Segovia, which had been
deteriorating since its initial construction nearly 2000 years earlier.
• In 1985, the United Nations declared the Roman Aqueduct of Segovia to be a Heritage of
Humanity and prompted the Spanish government to begin renovations on the aqueduct, which
had been deteriorating since its initial construction nearly 2000 years earlier.
• In 1985, the United Nations declared the Roman aqueduct of Segovia a Heritage of Humanity,
prompting the Spanish government to begin renovations on the aqueduct, which had been
deteriorating since its initial construction nearly 2000 years earlier.
The original sentence contains a misplaced modifier, which alters the intended
meaning of the sentence. The modifying phrase “Nearly 2000 years after its initial
construction” incorrectly modifies “the United Nations,” the adjacent noun. However, it
is the “Roman aqueduct” that was constructed nearly2000 years earlier, not “the
United Nations.” Further, “declared the Roman aqueduct…to be a Heritage of
Humanity” uses an incorrect idiom: “declare X to beY.” The correct form of the idiom
is: “declare X Y.”
(A) This choice is incorrect as it repeats the original sentence.
(B) This sentence implies that it was the deterioration of the aqueduct that prompted
the “Spanish government to begin renovations.” However, the intended meaning, as
dictated by the original sentence, is that the United Nations’ declaration prompted the
renovations. Further, “declared the Roman aqueduct…to be a Heritage of Humanity”
uses an incorrect idiom: “declare X to be Y.” The correct form of the idiom is: “declare
X Y.”
(C) The modifying phrase “After being declared…in 1985” incorrectly modifies the
adjacent noun “Spanish government.” It is not the “Spanish government” that was
declared a Heritage of Humanity, but rather the “Roman aqueduct.” Additionally, the
modifying phrase “which had been deteriorating…” incorrectly modifies the
immediately preceding noun, “Segovia.” Again, it was not “Segovia” that had been
deteriorating, but rather the “Roman aqueduct.”
(D) The verbs “declared” and “prompted” are written with parallel structure.
This changes the original meaning of the sentence by making these actions
independent and sequential. However, the intended meaning is that the
“prompting” occurred not independently of the declaration, but as a
consequence of the declaration. Further, “declared the Roman aqueduct…to
be a Heritage of Humanity” uses an incorrect idiom: “declare X to be Y.” The
correct form of the idiom is: “declare X Y.”
(E) CORRECT. This sentence is clear in meaning. The modifying phrase “which had
been deteriorating…” correctly modifies the immediately preceding noun “aqueduct.”
Also, the phrase “prompting the Spanish government…” is subordinate to “declared,”
making it clear that the “prompting” occurred as a result of the declaration. Finally,
“declared the Roman aqueduct…a Heritage of Humanity” uses the correct form of the
idiom: “declared X Y.”

4. Geologists once thought that the molten rock known as lava was an underground remnant of
Earth's earliest days, sporadically erupting
through volcanoes, but they now know that it is
continuously created by the heat of the radioactivity deep inside the planet.

• was an underground remnant of Earth's earliest days, sporadically erupting
• had been an underground remnant of Earth's earliest days and sporadically erupted
• was an underground remnant of Earth's earliest days, which sporadically erupted
• would be an underground remnant of Earth's earliest days that sporadically erupted
• was an underground remnant of Earth's earliest days, having sporadically erupted
The original sentence does not contain any errors. The verb clause "was an
underground remnant of Earth's earliest days" is correct in tense (simple past
"was") and number (singular "molten rock" paired with singular "was").
Moreover, the modifier "sporadically erupting through volcanoes" correctly
modifies "an underground remnant of Earth's earliest days."
•(A) CORRECT. This choice is correct as it repeats the original sentence.
•(B) This choice unnecessarily and incorrectly changes the simple past "was" to
the past perfect "had been," which is used only when describing the earlier of
two past actions. Moreover, the use of "and" here equates the geologists' false
understanding of lava with the fact that it sometimes erupts through volcanoes.
•(C) This choice improperly uses the relative pronoun "which" to modify "Earth's
earliest days" instead of "molten rock known as lava."
- 78 -
•(D) This choice incorrectly changes the simple past "was" to the conditional
"would be." Moreover, the use of "that" implies that eruption through volcanoes
was part of what the geologists erroneously believed about lava.
•(E) In this choice, "having sporadically erupted" incorrectly places this modifier
in the past tense, implying that lava no longer erupts through volcanoes.

5. Disease, pollution, and overfishing have devastated the bountiful oyster harvests that once
sustained the residents of the Chesapeake Bay area.

• of the Chesapeake Bay area
• in and around the Chesapeake Bay
• of the Chesapeake Bay
• around the vicinity of the Chesapeake Bay
• living in and around the Chesapeake Bay area
The given sentence is correct as written. "The residents of" a certain place is the
proper idiom. It is also correct to refer to the residents living in the "area" of the
Chesapeake Bay, rather than in the Bay itself.
•(A) CORRECT. The original sentence is correct as written.
•(B) This answer incorrectly implies that the residents are living "in" the Bay
itself as well as the area surrounding the Bay. (Note that if we were talking
about residents with houseboats or the like, they would be living "on" the Bay,
not "in" it.)
•(C) This answer implies that the residents reside only in or on the Bay itself
rather than near it or around the Bay area; though there may be some
residents living on boats, the meaning of the original sentence indicates it was
not intended to be limited to those living in or on the Bay. In addition, logic
dictates that the residents cannot live "in" the Bay.
•(D) "Around the vicinity of" is both redundant and the incorrect idiom; to live in
the "vicinity" of a landmark already includes the area "around" that landmark.
The correct idiom is "in the vicinity of."
•(E) "Living in and around the Chesapeake Bay area "is redundant; living "in" a
particular "area" implies living "around" that same area.

6. The spending on durable goods like household appliances and automobiles is a cyclical pattern
that depends on if the overall economy is healthy, whereas non-durable goods like food and
shelter remain constant regardless of the economy
.

• The spending on durable goods like household appliances and automobiles is a cyclical pattern
that depends on if the overall economy is healthy, whereas non-durable goods like food and
shelter remain constant regardless of the economy.
• Regardless of the economy, spending on non-durable goods like food and shelter remains
constant even though spending on durable goods like household appliances and automobiles is
a cyclical pattern that depends on whether the overall economy is healthy.
• Spending on durable goods, such as household appliances and automobiles, follows a cyclical
pattern that depends on the health of the overall economy, whereas spending on non-durable
goods such as food and shelter remains constant regardless of the economy's health.
• Whether the overall economy is healthy determines the cyclical pattern of spending on durable
goods such as household appliances and automobiles, whereas non-durable spending such as
food and shelter remains constant regardless of the economy.
• The cyclical pattern of spending on durable goods such as household appliances and
automobiles depends on whether the overall economy is healthy but non-durable goods like
food and shelter remain constant regardless of the economy.
The original sentence contains several errors. First, "household appliances and
automobiles" are specific examples of durable goods, so they ought to be
introduced with "such as" instead of "like." Similarly, "food and shelter" are specific
examples of non-durable goods, so “like” is used incorrectly there, too. Second,
the use of "if" in this context is incorrect. On the GMAT, "if" is used only to
introduce conditional clauses (e.g. “if X, then Y”). Here, the author should have
used "whether" instead of “if” to indicate uncertainty about the health of the overall
economy. Third, it is illogical to say that "spending…is a cyclical pattern". The
author clearly means that spending follows a cyclical pattern. Finally, the author’s
intent is to make a comparison between spending on durable goods and spending
on non-durable goods, but the original sentence incorrectly compares “spending
on durable goods” to “non-durable goods.”
(A) This choice is incorrect as it repeats the original sentence.
(B) First, "household appliances and automobiles" are specific examples of
durable goods, so they ought to be introduced with "such as" instead of "like."
Similarly, "food and shelter" are specific examples of non-durable goods, so “like”
is used incorrectly there, too. Additionally, it is illogical to say that "spending…is a
cyclical pattern". The author clearly means that "spending" follows a "cyclical
pattern."
(C) CORRECT. The specific examples of durable and non-durable goods are
correctly introduced with “such as.” The comparison is made in a logically and
structurally parallel way: “spending on durable goods…follows a cyclical pattern” is
parallel to “spending on non-durable goods…remains constant.”
(D) The phrasing of this choice is wordy and awkward, and “determines the
cyclical pattern of spending on durable goods” is not structurally parallel to “non-
durable spending … remains constant.” Finally, “non-durable spending” has a
nonsensical meaning; it is the goods that are non-durable, and the author’s intent
was to refer to spending on such goods. NOT parallel.
•(E) "Food and shelter" are specific examples of non-durable goods, so they
ought to be introduced with "such as" instead of "like." Also, this choice states
that "non-durable goods...remain constant" when what is meant is that
"spending on non-durable goods...remains constant.”

7. One characteristic of top-performing sales organizations is that they have a tendency to have
concentrated greater resources in the direction of a smaller, more careful selection of a
number of important customers than is the case with
other sales organizations.

• that they have a tendency to have concentrated greater resources in the direction of a smaller,
more careful selection of a number of important customers than is the case with
• that they tend to concentrate more resources to a smaller, more careful selection of a number
of important customers than toward
• that they have a tendency to concentrate more resources on a smaller, more careful selection
of a number of important customers as opposed to
• that they tend to concentrate more resources on a smaller, more carefully selected number of
important customers than do
• the tendency to concentrate a greater amount of resources on a careful and small selection of
a number of important customers as opposed to
The original sentence correctly compares a characteristic of top-performing sales
organizations with that of other sales organizations. However, the original sentence is
unnecessarily wordy in its use of “they have a tendency” as well as “in the direction
of” and “is the case.” Moreover, the use of the present perfect verb construction “have
concentrated” is inappropriate, since the simple present tense is sufficient to describe
a regular feature of “sales organizations.”
(A) This choice is incorrect as it repeats the original sentence.
(B) This choice is clear and concise. However, in its use of “toward other sales
organizations,” this choice does not draw the correct and logical comparison between
the behavior of top sales organizations and the behavior of other sales organizations.
Instead, this choice illogically compares the level of resources concentrated on
certain important customers and the resources directed toward other sales
organizations. Finally, the construction “concentrate more resources to” is
unidiomatic; the appropriate idiom is “to concentrate on.”
(C) This choice incorrectly draws a comparison between the level of resources
concentrated on a number of important customers and the resources directed
towards other sales organizations in its use of “as opposed to other sales
organizations.” The correct comparison is between the behavior of top sales
organizations and that of other sales organizations.
(D) CORRECT. This choice correctly draws a comparison between a characteristic of
top sales organizations and that of other sales organizations, and is otherwise clear
and concise.
(E) This choice incorrectly draws a comparison between the level of resources
directed toward a number of important customers and the resources directed toward
other sales organizations in its use of “as opposed to.” The correct comparison is
between top sales organizations and other sales organizations.

8. When airline carriers are able to impose a significant fare increase without deterring many
price-sensitive passengers
, it is an encouraging sign for the health of the airline industry.

• When airline carriers are able to impose a significant fare increase without deterring many
price-sensitive passengers
• When airline carriers are able to impose a significant fare increase without deterring many
passengers who are price-sensitive
• When airline carriers may raise fares significantly without it acting as a deterrent to many
price-sensitive passengers
• When airline carriers may raise fares significantly without it deterring many price-sensitive
passengers
• When airline carriers are able to impose a significant fare increase without it deterring many
price-sensitive passengers
In C, D and E, the two it’s have to mean the same thing, which is not the case.
•The meaning of the original sentence is clear: If passengers are not deterred
by a significant fare increase, the airline industry must be doing well. The
original sentence also uses concise language ("price-sensitive passengers") to
make its point. Additionally, the pronoun "it" in the original sentence clearly
refers to the fact that passengers are not deterred by a significant fare
increase.
•(A) CORRECT. This choice is correct as it repeats the original sentence.
•(B) This choice incorrectly replaces the concise phrase "price-sensitive
passengers" with the wordy alternative "passengers who are price-sensitive."
•(C) In this choice, the pronoun "it" is used initially to refer to a fare increase. In
the non-underlined portion of the sentence, a second "it" is used to refer not to
a fare increase, but to the fact that a fare increase does not deter price-
sensitive passengers. The use of the pronoun "it" is incorrect in this answer
choice as it causes the antecedent to be unclear for the second "it" in the non-underlined portion of the sentence. Also, "acting as a deterrent" is
unnecessarily wordy, and the use of the term "may raise" suggests that the
airlines are being given permission to increase their fares.
•(D) In this choice, the pronoun "it" is used initially to refer to a fare increase. In
the non-underlined portion of the sentence, a second "it" is used to refer not to
a fare increase, but to the fact that a fare increase does not deter price-sensitive passengers. The use of the pronoun "it" is incorrect in this answer
choice as it causes the antecedent to be unclear for the second "it" in the non-underlined portion of the sentence. Also, the use of the term "may raise"
suggests that the airlines are being given permission to increase their fares.
•(E) In this choice, the pronoun "it" is used initially to refer to a fare increase. In
the non-underlined portion of the sentence, a second "it" is used to refer not to
a fare increase, but to the fact that a fare increase does not deter price-sensitive passengers. The use of the pronoun "it" is incorrect in this answer
choice as it causes the antecedent to be unclear for the second "it" in the non-underlined portion of the sentence.

9. The pioneering research of Lewis Latimer and Thomas Edison, who became known for his
invention of the light bulb
, accelerated the development of the first power plant, which opened
in New York City in 1882.

• of Lewis Latimer and Thomas Edison, who became known for his invention of the light bulb,
• of Lewis Latimer and Thomas Edison, known for his invention of the light bulb,
• of Thomas Edison, known for his invention of the light bulb, and Lewis Latimer
• of Lewis Latimer and Thomas Edison became known for his invention of the light bulb and
• that was conducted by Thomas Edison, who became known for his invention of the light bulb,
and Lewis Latimer
In the original sentence, the pronoun “his” lacks a clear antecedent, making it
unclear whether it was “Lewis Latimer” or “Thomas Edison” who “became
known for his invention of the light bulb.” In fact, the plural phrase "Lewis
Latimer and Thomas Edison" leads us to expect a plural pronoun later on; if we
only wish to refer to "Thomas Edison," we should position the modifying phrase
so as to refer to "Thomas Edison" only.
Also, the construction “who became known for his invention” is wordy and could be
replaced by the more concise form “known for his invention.”
(A) This choice is incorrect as it repeats the original sentence.
(B) This answer choice replaces the wordy construction “who became known for his
invention” with the more concise form “known for his invention,” but retains the
original ambiguity stemming from the lack of a clear antecedent for the pronoun “his.”
(C) CORRECT. By placing the modifier “known for his invention of the light bulb”
immediately after “Thomas Edison” and prior to the introduction of “Lewis Latimer,”
this answer choice resolves the original ambiguity and makes it clear that the pronoun
“his” refers to “Thomas Edison” rather than “Lewis Latimer.” The construction “known
for his invention” is also more concise than the original form “who became known for
his invention.”
(D) This answer choice illogically states that it was the “pioneering research” rather
than “Thomas Edison” that became “known for his invention of the light bulb,” thus
altering the original meaning of the sentence. Further, this answer choice retains the
original problem of ambiguity by failing to provide a clear antecedent for the pronoun
“his.”
(E) This answer choice uses the passive construction “research that was conducted
by Thomas Edison” rather than the more direct and concise form “research of
Thomas Edison.” Further, while the placement of the modifier “who became known for
his invention of the light bulb” next to Thomas Edison and prior to the introduction of
“Lewis Latimer” resolves the ambiguity, the phrase “who became known for his
invention” is wordy; the more concise form “known for his invention” is preferable.

10. The bowerbirds of Australia derive their name from the fact that the males build elaborate
bowers of sticks and twigs to attract females, decorating them with flowers and other
vegetation
in a display of courtship.

• the fact that the males build elaborate bowers of sticks and twigs to attract females,
decorating them with flowers and other vegetation
• the elaborate bowers of sticks and twigs that the males build and decorate with flowers and
other vegetation in order to attract females
• the elaborate bowers of sticks and twigs, decorated with flowers and other vegetation that the
males use to attract females
• the fact that the males build elaborate bowers of sticks and twigs, having decorated them with
flowers and other vegetation, to attract females
• the elaborate bowers of sticks and twigs that are built by the males and decorated with
flowers and other vegetation to attract females
The original sentence contains the pronoun "them" but it is not grammatically clear
whether the pronoun's antecedent is "bowers of sticks and twigs" or
"females." Logically, we know that the antecedent is "bowers", so we need to find a
replacement that makes this clear. Moreover, the bowerbird does not derive its name
from the fact that it builds bowers, but from the bowers themselves.
(A) This choice is incorrect as it is the same as the original sentence.
(B) CORRECT. This choice rewrites the sentence to make it clear that the name
derives from the bowers and not from the fact of building them, and it also eliminates
the pronoun "them" and instead refers to "structures" to make the relationship clear.
(C) This choice does not make it clear that the males build the bowers and decorate
them. Instead, it seems to suggest that the bowers exist on their own and that the
male uses only the flowers and vegetation to attract females.
(D) This choice uses the phrase "having decorated them" improperly. It is not
necessary to use "having" in this context because the sentence describes an ongoing
event, not one that occurred in the past.
(E) This choice is in the passive voice, which is not preferable to active voice when a
grammatical active version (such as B) is also offered. Moreover, the placement of
the modifier "that are built by the males" incorrectly implies the sticks and
twigs are built by the males. Also the phrase "and decorated with flowers and other
vegetation to attract females" seems to further imply that the sticks and twigs are also
decorated with flowers...

11. Although reclusive author Harper Lee wrote just one book in her lifetime and that book is
widely considered a masterpiece.

• lifetime and that book
• lifetime and it
• lifetime, that book
• lifetime; it
• lifetime; that book
The initial connecting word "although" indicates a change of direction will occur later
in the sentence, but the subsequent connecting word "and" incorrectly allows the
sentence to continue in the same direction instead of introducing a contrast.
(A) This choice is incorrect because it repeats the original sentence.
(B) This choice also uses the incorrect connecting word "and" when the initial
"although" indicates a change of direction is necessary. In addition, "it" has an
unclear antecedent; it could be referring back to "book" or "lifetime."
(C) CORRECT. This choice correctly removes the connecting word "and," enabling
the change of direction indicated by "although" to take place successfully.
(D) This choice uses a semicolon incorrectly. Semicolons require each clause before
and after the semicolon to be complete sentences, and here "Although reclusive
author Harper Lee wrote just one book in her lifetime" is not a complete sentence. In
addition, "it" has an unclear antecedent; it could be referring back to "book" or
"lifetime."
(E) This choice uses a semicolon incorrectly. Semicolons require each clause before
and after the semicolon to be complete sentences, and here "Although reclusive
author Harper Lee wrote just one book in her lifetime" is not a complete sentence.

12. Some scientists suggest the moon had been formed out of part of the Earth, which was
dislodged perhaps
by a meteor.

• the moon had been formed out of part of the Earth, which was dislodged perhaps
• that the moon was formed from part of the Earth that had perhaps been dislodged
• that part of the Earth formed the moon, which was dislodged perhaps
• the moon was formed out of part of the Earth, having perhaps been dislodged
• that the moon had been formed from part of the Earth, which perhaps had been dislodged
There are several errors in the original sentence. First, “some scientists
suggest the moon…” illogically indicates that the moon is the object of the verb
“suggest.” The scientists are not suggesting the moon, rather they are
suggesting something about the moon. Second, “formed out of” is wordier than
the preferred idiom “formed from.” Finally, the relative pronoun “which” must
refer to the immediately preceding noun, suggesting illogically in this case that
“the Earth” was dislodged by a meteor. It is more likely that the author intends
to say that “a part” of the Earth was dislodged, or that “the moon” was
dislodged from the Earth.
(A) This choice is incorrect as it repeats the original sentence.
(B) CORRECT. This choice begins with “some scientists suggest that the
moon was formed…” clearing up the confusion from the original sentence
about what the scientists are suggesting. Second, this choice uses the
preferred idiom “formed from.” The modifying phrase “that had perhaps been
dislodged” correctly refers to “part of the Earth.”
(C) The use of the active verb “formed” is incorrect here because it illogically
suggests that “part of the Earth” had an active role in forming the moon.
(D) First, “some scientists suggest the moon…” illogically indicates that the
moon is the object of the verb “suggest.” The scientists are not suggesting the
moon, rather they are suggesting something about the moon. Second, “formed
out of” is wordier than the preferred idiom “formed from.”
(E) The scientists suggest that two actions occurred: “the moon had been
formed” and “part of the Earth…had been dislodged. ”Both of these actions
took place in the distant past, and it is logical to infer that the part was
dislodged, and later the moon was formed from it. However, this choice uses
the past perfect tense for both actions, incorrectly indicating that the part was
dislodged and the moon simultaneously formed. Furthermore, the past perfect
tense is only used correctly to indicate that one action took place prior to some
other action in the simple past tense; this sentence has no verbs in the simple
past tense, so the use of the past perfect tense is not warranted.

13. The work of Byron and Shelley, like other poets of their era, explored themes of love and
beauty, which gave rise to the school of poetry known as Romanticism
.

• The work of Byron and Shelley, like other poets of their era, explored themes of love and
beauty, which gave rise to the school of poetry known as Romanticism.
• Byron and Shelley, like other poets of their era, explored themes of love and beauty in their
work, giving rise to the school of poetry known as Romanticism.
• Like other poets of their era, Byron and Shelley's work explored themes of love and beauty,
giving rise to the school of poetry known as Romanticism.
• Love and beauty are themes explored by the work of Byron and Shelley, like they were by
other poets of the era, and they gave rise to the school of poetry known as Romanticism.
• The school of poetry known as Romanticism rose from the works of Byron and Shelley, which
was like that of other poets of the era in exploring themes of love and beauty.
The original sentence incorrectly compares the work of Byron and Shelley to poets. In
addition, the use of "which" in the original sentence incorrectly implies that "themes of
love and beauty" gave rise to Romanticism.
(A) This choice is the same as the original sentence.
(B) CORRECT. The comparison is correctly drawn between Byron and Shelley and
"other poets." Moreover, the original problematic use of "which" has been corrected.
(C) This choice incorrectly compares "other poets" to "Byron and Shelley's work."
(D) This choice incorrectly uses "like" to compare verb clauses. Instead, "as" would
be appropriate here. Moreover, parallelism requires that the comparison be made
between "the work of Byron and Shelley" and "the work of other poets" or between
"Byron and Shelley" and "other poets." Instead, we have "the work of Byron and
Shelley" and "other poets."
(E) This is choice is awkward and wordy. Moreover, the verb "was" incorrectly refers
to "the works of Byron and Shelley."

14. Quarried from a site over five miles away, scientists are still puzzled as to how the prehistoric
Britons managed to transport the massive stone blocks of Stonehenge over such a great
distance without machinery
.

• scientists are still puzzled as to how the prehistoric Britons managed to transport the massive
stone blocks of Stonehenge over such a great distance without machinery.
• the massive stone blocks of Stonehenge are still puzzling to scientists because of how the
ancient Britons managed to transport them over such a great distance without machinery.
• scientists are still puzzled by how the prehistoric Britons managed the transportation of the
massive stone blocks of Stonehenge without machinery over such a great distance.
• the massive stone blocks of Stonehenge still puzzle scientists, who wonder how the prehistoric
Britons managed to transport them over such a great distance without machinery.
• the massive stone blocks of Stonehenge are still a puzzle to scientists due to being transported
over such a great distance without machinery.
The sentence begins with a modifier: "quarried from a site over five miles away". This
clearly describes stone. However, the subject of the modifier in the original sentence
is "scientists." This is incorrect. We need to find a choice that places some kind of
stone as the subject of the modifier.
(A) This choice is the same as the original.
(B) While the opening modifier correctly modifies "the massive stone blocks," the
phrase "because of how" seems to imply that the prehistoric Britons' method of
transporting the stones is known.
(C) This choice incorrectly uses "scientists" as the subject of the opening modifier.
(D) CORRECT. "Massive stone blocks" is correctly placed as the subject of the
modifier.
(E) While the opening modifier correctly modifies "the massive stone blocks," this
sentence omits reference to the prehistoric Britons and contains the awkward phrase
"due to being transported".
General Discussion
Manager
Manager
Joined: 11 May 2011
Posts: 235
Own Kudos [?]: 212 [1]
Given Kudos: 84
Send PM
Re: GMATCLUB VERBAL ATTACK [#permalink]
1
Bookmarks
My answers are listed below. I took 24 min for all of them.
C D C E C C B D E B.
I got 2 of them wrong. One wrong answer is acceptable but other one is not.
Also, we must discuss question 4 because i'm not happy with the explanation floating around for that one.

Cheers!
Alum
Joined: 19 Mar 2012
Posts: 4341
Own Kudos [?]: 51447 [9]
Given Kudos: 2326
Location: United States (WA)
Concentration: Leadership, General Management
Schools: Ross '20 (M)
GMAT 1: 760 Q50 V42
GMAT 2: 740 Q49 V42 (Online)
GMAT 3: 760 Q50 V42 (Online)
GPA: 3.8
WE:Marketing (Non-Profit and Government)
Send PM
Re: GMATCLUB VERBAL ATTACK [#permalink]
4
Kudos
5
Bookmarks
Expert Reply

STRENGTHEN SET OFFICIAL EXPLANATIONS


Answer KEYS
CDCBCCBDEC

1.
The conclusion of the argument is that the government's calculation methods
must be altered in order to provide statistics that measure true poverty. To
support this position, the author first explains how the government’s method
works and then introduces a hypothetical example that would return a "false
positive" - that is, a person who has a large income, yet is classified by the
government as living in poverty. One example, however, is generally not enough
to invalidate an entire method; no method is perfect and there are always a few
results that are not consistent with the overall conclusion. In order to validate, or
strengthen, the conclusion, we need to show that the government’s method is
fundamentally inferior to some alternative that would produce more valid results.

(A) This choice weakens the argument by minimizing the importance of the
author's evidence (the hypothetical retiree with capital gains). According to this
choice, the use of cash income to designate poverty levels is a very sound
method because it provides valid results for more than 99% of those classified as
living in poverty.

(B) This choice shows that the government’s method provided a wide range of
results for the poverty rate over a certain period of time, but it is irrelevant to the
argument at hand. It tells us nothing about whether the method provides relevant
statistics in any given year.

(C) CORRECT. If this statement is true, then the government’s calculation
method seems to overstate the number of people living in poverty, while the
various private sector studies generally agree with each other that the number of
people is lower. Thus, the methods used in the private sector are likely to be
more valid than the government’s method, lending credence to the author's
contention that the government’s method should change.

(D) Although this choice provides an example of people who might agree with the
conclusion (several prominent economists), this choice provides no evidence that
the alternate method they endorse would provide more relevant statistics than
the government’s method.

(E) This choice adds another hypothetical example of how the current method
could include someone in the poverty count who does not actually live in poverty.
It does not, however, address whether there are other calculation methods that
are more accurate than the government’s method.

2.
The public health advocates are concerned that patients are subjected to
advertisements about prescription drugs, and may pursue these drugs even
though the drugs may not be clinically appropriate. It is argued that, because
physicians must prescribe the drugs in question, patient pursuit of these
prescription drugs is irrelevant. However, patients who pursue and request
particular prescription drugs may be able to encourage or induce a physician to
prescribe drugs that he or she might not have in the absence of such
encouragement.

(A) The clinical efficacy of certain over-the-counter medications does not address
the public health advocates’ concern regarding patient pursuit of inappropriate
prescription drugs.

(B) The public health advocates’ concern does not rely on every possible
consumer to see the advertisements for prescription medications. It is enough
that some consumers see the advertisements.

(C) This answer choice states the obvious possibility that physicians may also
see the advertisements for prescription drugs directed toward consumers. This is
irrelevant to the concern expressed by the public health advocates.

(D) CORRECT. This answer choice directly addresses the public health
advocates’ concern by establishing that physicians are not susceptible to patient
pressure in prescribing inappropriate drugs. As a result, drugs will be prescribed
according to the objective clinical judgment of the prescribing physician,
mitigating the danger of inappropriate use.

(E) That certain prescription medications are safe and effective treatments for
many conditions does not address the concern of inappropriate use expressed
by the public health advocates. Prescription drugs can be safe and effective
when used for certain conditions by various individuals and still be subject to
unhealthy use by other patients.

3.
This argument concerns a potential explanation for larger tips on the part of
restaurant patrons. The explanation provided is that customers are more
generous toward servers that leave their hand-written name on the bill due to a
greater degree of personal identification with the server, which encourages larger
tips. The correct answer will either support the fact that a hand-written name
strengthens personal identification, or that personal identification encourages
larger tips.

(A) The fact that the effect applies equally regardless of the method of payment
is not relevant to the conclusion.

(B) The argument does not address the size of the bill; rather, it addresses the
size of the tip. This choice is irrelevant.

(C) CORRECT. This answer choice provides further evidence that a hand-written name or signature generates a greater form of personalization and
emotional connection among recipients, leading to more donations.

(D) The impact of alcoholic beverages on tipping behavior is irrelevant.

(E) The location of the restaurants and leisure pursuits of the patrons are
irrelevant to the argument.

4.
The conclusion is that a company should wait until purchases of an old device
have begun to decline before announcing a new device. The basis for this claim
is that consumers stop buying the old device. We are asked to strengthen the
argument.

(A) The typical drop in the price of new technology does not influence whether a
company should wait until sales of an old technology begin to decline before
introducing a new one.

(B) CORRECT. This choice states that media outlets such as television and
magazines often report on the planned introduction of new devices while sales of
old devices are still strong. The argument requires that consumers "hear about
the new device"; stories in the media provide a means for consumers to do so.

(C) If many consumers are unable to determine the superiority of new
technology, then they might be less swayed to wait for a new, recently
announced device than otherwise. As a result, this claim may be seen to weaken
the analyst's assertion.

(D) The number of technology purchases per year does not directly relate to this
argument. The argument is about waiting until the consumer demand declines
before announcing a new technology. However frequently consumers typically
purchase technology, some will be ready to buy the old device when news of the
upcoming device gets out -- and according to the argument, this news will cause
some of those consumers to wait.

(E) The passage makes no mention of whether the technologies belong to the
same company or different companies.

5.
The argument claims that federal incentives should be provided to encourage
energy efficiency. The argument also notes that companies are already working
in this direction and that this trend will ease the environmental and energy
pressures that currently trouble the world. Supporting this argument could involve
providing evidence of possible success for these efforts toward their goals.

(A) This choice is an irrelevant comparison. That Canadian companies are more
efficient has no bearing on efforts in the United States or the role of government
incentives.

(B) This choice does not strengthen the claim. Experts' claims are not the same
as reality. Furthermore, the choice does not say whether reducing energy use to
the 1995 level is a significant decrease, nor does it provide any information to
strengthen the link between government incentives and reduced energy use.

(C) CORRECT. This choice provides evidence that government incentives are
effective. Thus, this choice confirms an assumption that the conclusion is
feasible.

(D) This choice is an irrelevant distinction. It does not matter to the argument’s
conclusion if one of these issues is a greater problem than the other in the
present.

(E) The passage asserts that the United States should be at the forefront of an
emerging market for cleaner technologies; i.e., the market will be significant in
the future. The size of the market at present is irrelevant to the argument.


6.
The director concludes that the fee hike has helped to counteract the cut in state
funding. In other words, the director believes that increasing the late fees has led
to increased revenue from late fees. While the size of the fee itself is one
important factor, there are other factors that also have an effect on the amount of
revenue generated from late fees: the number of overdue books and the number
of days that books are overdue before they are returned. It’s very possible that
the fee increase would prompt more borrowers to return their books on time; this
would reduce the number of late fees being paid, reducing revenue from late
fees. Further, it’s possible that the fee increase would prompt more borrowers to
return their already overdue books sooner than they would otherwise. This would
reduce the average amount of each late fee, reducing revenue from late fees.
The argument explicitly states that there has been no decline in the number of
overdue books, but it says nothing about the number of days that books are
overdue before they are returned. A statement that rules out the possibility that
borrowers are returning their already overdue books sooner than they would
have if they were still being charged the original lower overdue fee would
strengthen the director’s claim.

(A) A decrease in the number of borrowed books has no bearing on the revenue
generated from late fees if the number of overdue books remains unchanged.
The question explicitly states that the number of overdue books has not changed.

(B) If anything, this statement weakens the argument. The costs incurred to
implement the new fees would cut into the revenue generated from these new
fees.

(C) CORRECT. This statement rules out the possibility that the library system is
losing revenue as a result of borrowers returning overdue books earlier than they
would otherwise.

(D) The argument has explicitly stated that the number of overdue books has not
changed. This is a stated premise that we must take as factual information,
regardless of the quality of the database being used to track such information.

(E) The elimination of other unrelated costs has no bearing on whether the library
system has successfully increased revenues through late fees.


7.
The conclusion is that a developer who wishes to make a large profit would be
wise to buy urban waterfront lots and erect residential buildings on them. The
basis for that claim is that people pay large sums for beach front homes. We are
asked to strengthen this argument.

(A) This choice states that people have more buying power today than in
previous centuries. This does not strengthen the claim that a developer will make
money on urban waterfront properties.

(B) CORRECT. This choice states that homeowners will be willing to spend large
sums of money on residential properties in traditionally industrial or commercial
districts. Since we know from the argument that urban waterfronts have
traditionally been industrial, this fact strengthens the claim that a developer can
make a profit on urban waterfront properties.

(C) This choice states that many urban waterfront lots are available for purchase.
This does not suggest, however, that a developer will be able to sell them after
he or she builds on them.

(D) This choice states that many coastal cities are giving tax breaks to
developers who rehabilitate the waterfront. But this does not suggest that anyone
will buy the developed properties.

(E) This choice states that properties in the interior of cities are more expensive
than those on the waterfront. Although waterfront properties are therefore
cheaper to acquire, this does not necessarily mean that a developer can make a
profit after buying such properties.

8.
The question asks for information that will support the conclusion that students
attending charter schools will, on average, perform better on assessments of
writing ability than students attending traditional public schools. The passage
specifies that charter schools differ from non-charter public schools in that
charter schools have more freedom to innovate and that they are held
accountable for meeting specific educational outcomes. One way to support the
conclusion is to demonstrate that one of the two differences cited between
charter and non-charter public schools is somehow tied to higher performance on
writing assessments.

(A) While the passage mentions that charter schools themselves are freed from
many regulations, no information is presented about any difference in emphasis
with respect to order and discipline between charter and non-charter public
schools. As such, it is impossible to tell whether this information would support
the conclusion in the question.

(B) This choice presents information only about those students who score at the
very highest level of the writing assessments. However, this presents no
information about the difference, on average, between all charter school students
and non-charter public school students. It is possible, for example, that while the
students who perform at the highest level on writing assessments are those who
attend charter schools, on average non-charter public school students perform
better.

(C) There is no necessary link between the amount of time spent teaching writing
and student performance on writing assessments. For example, a good teacher
who spends one hour teaching writing may have a more positive impact on
student performance than a poor teacher who spends three hours teaching
writing.

(D) CORRECT. The passage specifies that charter schools have more freedom
to pursue innovative educational ideas than non-charter public schools. It follows
that charter schools are allowed to experiment with their curricula to a greater
degree than non-charter public schools. This choice links this difference to higher
student achievement on assessments of writing ability.

(E) The number of students attending charter vs. non-charter schools has no
bearing on the conclusion. The conclusion focused on student performance on
average, thereby eliminating raw numbers of students as relevant to this
measure.



9.
League officials plan to reduce the number of flagrant fouls by implementing
mandatory suspensions for players who commit such fouls. This plan will work
only if the punishment serves to deter players from committing flagrant fouls.

(A) The cause of injuries has no bearing on whether suspensions will deter
players from committing flagrant fouls.

(B) While the referees’ effectiveness in recognizing and reporting flagrant fouls
will surely aid in the implementation of the new policy, this has no bearing on
whether the policy will deter players from committing flagrant fouls.

(C) The parents’ opinion has no bearing on whether the suspensions will deter
players from committing flagrant fouls.

(D) While we might conclude that the other, similar league has a low incidence of
flagrant fouls because it suspends players who commit such fouls, we have no
evidence to show that the suspensions actually deter players from committing
fouls. It is entirely possible that the other league has a low incidence of flagrant
fouls for other reasons. For example, maybe the players in the other league are
just inherently less aggressive.

(E) CORRECT. If players want to make the All-Star team, and if a record of
suspension precludes these players from being selected for the team, then
players are less likely to commit fouls that will lead to suspensions.


10.
The passage makes the premise that microwave ovens are not completely safe.
This is followed by a conclusion by the consumer advocates that microwave
ovens should not be accepted as standard appliances. Since there is nothing in
the passage that provides an explicit link between the safety of microwave ovens
and their acceptability as standard appliances, the consumer advocates’
conclusion is based on an assumption (i.e., an implied premise) that “an
appliance should be accepted as standard only if it is found to be completely
safe.” The most effective way to strengthen such a conclusion is to show that
such an assumption is indeed true.

(A) The strength of the consumer advocates’ argument hinges upon the link
between the level of safety of microwave ovens and the rationale for their
acceptance in the home. Any lack of joy in microwave cooking is not relevant to
the argument.

(B) Providing a specific example of how a person might be injured, even
seriously, by a microwave oven may provide emotional support for the consumer
advocates’ position, but does little to strengthen the argument logically: the
possibility of injury has already been stipulated as a premise.

(C) CORRECT. This choice best strengthens the argument by making explicit the
assumption upon which the consumer advocates’ argument was based.

(D) If no appliance is completely safe, then the consumer advocates’ argument is
absurb: no appliance is, or ever will be, acceptable as “standard” in a modern
kitchen. This choice weakens the conclusion.

(E) The relative energy efficiency of gas vs. microwave cooking is not relevant to
this argument.


Please feel free to post particular question doubts, and I will be happy to clarify as much as I can.
Alum
Joined: 19 Mar 2012
Posts: 4341
Own Kudos [?]: 51447 [0]
Given Kudos: 2326
Location: United States (WA)
Concentration: Leadership, General Management
Schools: Ross '20 (M)
GMAT 1: 760 Q50 V42
GMAT 2: 740 Q49 V42 (Online)
GMAT 3: 760 Q50 V42 (Online)
GPA: 3.8
WE:Marketing (Non-Profit and Government)
Send PM
Re: GMATCLUB VERBAL ATTACK [#permalink]
Expert Reply
Capricorn369 wrote:
My answers are listed below. I took 24 min for all of them.
C D C E C C B D E B.
I got 2 of them wrong. One wrong answer is acceptable but other one is not.
Also, we must discuss question 4 because i'm not happy with the explanation floating around for that one.

Cheers!


Please let me know which OA do you find debatable?
BTW All questions in this set are taken from MGMAT
Manager
Manager
Joined: 11 May 2011
Posts: 235
Own Kudos [?]: 212 [0]
Given Kudos: 84
Send PM
Re: GMATCLUB VERBAL ATTACK [#permalink]
souvik101990 wrote:
Capricorn369 wrote:
My answers are listed below. I took 24 min for all of them.
C D C E C C B D E B.
I got 2 of them wrong. One wrong answer is acceptable but other one is not.
Also, we must discuss question 4 because i'm not happy with the explanation floating around for that one.

Cheers!


Please let me know which OA do you find debatable?
BTW All questions in this set are taken from MGMAT


I would like to discuss question number 4.
I'm not satisfied with the given OA/OE. Can someone illustrate?

Cheers!
Alum
Joined: 19 Mar 2012
Posts: 4341
Own Kudos [?]: 51447 [1]
Given Kudos: 2326
Location: United States (WA)
Concentration: Leadership, General Management
Schools: Ross '20 (M)
GMAT 1: 760 Q50 V42
GMAT 2: 740 Q49 V42 (Online)
GMAT 3: 760 Q50 V42 (Online)
GPA: 3.8
WE:Marketing (Non-Profit and Government)
Send PM
Re: GMATCLUB VERBAL ATTACK [#permalink]
1
Kudos
Expert Reply
Capricorn369 wrote:
souvik101990 wrote:
Capricorn369 wrote:
My answers are listed below. I took 24 min for all of them.
C D C E C C B D E B.
I got 2 of them wrong. One wrong answer is acceptable but other one is not.
Also, we must discuss question 4 because i'm not happy with the explanation floating around for that one.

Cheers!


Please let me know which OA do you find debatable?
BTW All questions in this set are taken from MGMAT


I would like to discuss question number 4.
I'm not satisfied with the given OA/OE. Can someone illustrate?
Cheers!


4. Analyst: The pace of technological development brings a constant stream of new devices to the market, and many
of them enjoy commercial success. But announcing new technology too soon after the introduction of a successful
device can backfire. Once consumers hear about the new device, they may stop buying the one currently on sale.
So, if a company wishes to announce the upcoming sale of a new device, it should wait until purchases of the old
device have begun to decline. Which of the following, if true, would best support the analyst’s main assertion?
• New technology often becomes less expensive after an initial surge in sales.
• Media outlets, such as television programs and magazines, often report on the planned introduction of new
devices while the sales of old devices are still strong.
• Many consumers are unable to determine whether new technology is superior to current technology.
• Surveys have shown that some consumers make only one or two technology purchases per year, whereas others
make more frequent purchases.
• Consumers tend to be loyal to technology companies whose products they enjoy using.

Conclusion of this argument
So, if a company wishes to announce the upcoming sale of a new device, it should wait until purchases of the old
device have begun to decline

A. This has almost nothing to do with the argument as this is talking about the sales after the announcement and the release of the new device.
B. This seems to strengthen because if, according to the author, the company does not announce the new device, the media will have nothing to report and sales of the old device will remain strong.
C. As long as their indecisiveness doesn't hamper their buying tendencies, do we need to care?
D.This means almost everything. It means some people may buy the device while some people wont. In any case, we can draw much from this statement.
E. Loyalty can be proved both by purchasing new stuffs from the same company or by not purchasing anything at all(from that or any other company).

Hope this helps
Alum
Joined: 19 Mar 2012
Posts: 4341
Own Kudos [?]: 51447 [8]
Given Kudos: 2326
Location: United States (WA)
Concentration: Leadership, General Management
Schools: Ross '20 (M)
GMAT 1: 760 Q50 V42
GMAT 2: 740 Q49 V42 (Online)
GMAT 3: 760 Q50 V42 (Online)
GPA: 3.8
WE:Marketing (Non-Profit and Government)
Send PM
Re: GMATCLUB VERBAL ATTACK [#permalink]
4
Kudos
4
Bookmarks
Expert Reply
Sorry for the delay guys, but I will be up with a SC project soon
Here are the OEs for the weaken set


"WEAKEN" SET OFFICIAL EXPLANATIONS


WEAKEN
1.
On average, the team hit more home runs playing in front of larger crowds than
in front of smaller crowds. The argument attributes this statistic to the motivation
that comes from playing in front of larger crowds. In order to undermine this
conclusion, look for another reason to explain why more home runs were hit in
front of larger crowds.
(A) The argument makes a claim about the collective behavior of the team. This
collective claim does not preclude certain individuals from hitting fewer home
runs in larger stadiums.
(B) The claim made in the argument is based on the size of the crowd in each
stadium. For whom the fans cheered is irrelevant to the argument.
(C) Similar to answer choice A, this choice cites one specific example of
contradictory information, while the argument is based on the average behavior
of the team throughout the entire season. The does not strongly undermine that,
on average, the team was motivated by larger crowds.
(D) CORRECT. This choice explains that the larger stadiums actually have
different dimensions from the smaller stadiums. In order to accommodate a
larger number of fans, the outfield walls are closer to the batters. Thus, it is very
possible that the greater number of home runs is due to the fact that the ball
does not have to travel as far in larger stadiums.
(E) The announcer’s opinion is not relevant to the argument, and, even if it were,
this choice would strengthen the argument.
2.
The conclusion is that "if major industries increase their capital reserves, the
employment rate will not decline in the future." Why? Because major industry did
not have capital reserves. The author assumes that having capital reserves is
sufficient to prevent a decline in the employment rate. We are asked to cast
doubt (i.e., weaken) the author's claim.

(A) Whether the drop in employment was foreseen does not relate to the core of
the argument, which is that capital reserves will prevent another decline in the
employment rate.

(B) The fact that some major industries had appreciable capital reserves does
not contradict the claim that an increase in these reserves would prevent a future
drop in employment rates.

(C) CORRECT. The author neglects to take into account the fact that other
factors, such as an increase in labor costs, could adversely affect the
employment rate. For example, if the cost of labor becomes prohibitively
expensive, even with increased reserves, the employment rate could decline.

(D) Legislation mandating a certain level of reserves does not contradict the
claim that increased reserves would prevent a drop in employment rates.

(E) The fact that the employment rate was more severe this year than last does
not contradict the claim that an increase in reserves would prevent a drop in the
employment rate.
3.
Farmers in developing countries claim that the global price of wheat is low
because American farmers produce too much of the grain. They also claim that
American farmers produce too much wheat because they have no incentive to
manage their crops, since the U.S. government will buy whatever wheat
American farmers cannot sell on the open market. We are asked to find a choice that
weakens the claims of the farmers in developing countries that removing the
American subsidy would cause the price of wheat to rise.
(A) That there are uses for wheat that is not eaten is irrelevant here. This does
not address any aspect of the farmers' claims.
(B) The fact that buyers of wheat can predict their needs in advance is irrelevant
here, because the text indicates that American farmers do not pay attention to
actual demand for wheat.
(C) In this argument, the global market for soybeans is irrelevant to the global
market for wheat, which is a different commodity with different demand, supply,
and pricing structures.
(D) CORRECT. The farmers assume that the sole cause of the wheat surplus is
the United States. This answer choice suggests that other countries would
modify their output to counterbalance any reduction on the part of the United
States, keeping prices constant instead of allowing them to rise.
(E) The price of another crop is largely irrelevant. Moreover, the fact that the
price of sorghum, a non-subsidized crop, is lower tends to support, rather than
weaken, the claims of the farmers.
4.
This argument concludes that “Hollywood studios have little chance of making
money this year by exporting their films worldwide,” based on the premise that
profits from piracy overseas rose sharply last year. The argument assumes that
no other relevant conditions have changed for the better since then. The question
asks for a choice that does not weaken the argument, which means that the
correct answer will either strengthen it or is irrelevant.
(A) This choice weakens the argument. It attacks the necessary assumption that
$500 million is a large enough amount to destroy Hollywood profits.
(B) This choice weakens the argument. It attacks the necessary assumption that
there would not be a heightened and effective law enforcement campaign against
piracy.
(C) This choice weakens the argument. It attacks the assumption that there are
no other ways for Hollywood studios to profit from export of its films than DVDs.
(D) CORRECT. This choice is irrelevant. The conclusion (and premise) concern
money. That all of the Academy Award-nominated films were pirated last year
does not increase or diminish the probability of Hollywood studios making money
this year. Thus, this does not weaken the argument.
(E) This choice makes the conclusion less likely by providing a premise that
suggests that this year’s profits internationally will rise.
5.
The analyst argues that the mayoral candidate who opposes the deportation plan
will win the governor’s race because 60% of city residents also oppose the plan.
The analyst assumes that a majority of residents will vote for this candidate
based on his position on illegal immigration. Any statement that calls this
assumption into question will weaken the argument. We are looking for the one
statement that does NOT call this assumption into question.
(A) This statement calls into question the assumption that voters will cast their
ballots based on the illegal immigration issue. Therefore, this statement
weakens the analyst's argument.
(B) CORRECT. This does not weaken the argument. In fact, if some of those
who support the plan are willing to reconsider, they may ultimately oppose the
original plan and decide to vote for the candidate who is also in opposition. If
anything, this would help justify the analyst's claim that the candidate who
opposes the plan will win the election.
(C) This statement calls into question the assumption that a majority of residents
will vote for the candidate who opposes the plan. If many of these residents are
not registered voters, they will not be able to vote, regardless of their position on
the immigration issue. This weakens the argument.
(D) This calls into question the assumption that the residents will vote based on
the illegal immigration issue. This statement shows that voters have a history of
voting for the incumbent despite his controversial position on important issues. It
is possible that the voters will again vote for the incumbent, even if he has taken
an unpopular position on the illegal immigration issue. This weakens the
argument.
(E) If just under 30% of the residents are illegal immigrants, it is likely that many
of the 60% in opposition to the plan are actually illegal immigrants themselves. If
these people can’t vote, it is less likely that the candidate who opposes the plan
will win.
6.
Adam concludes that to cover the $4,000 cost of a booth at the fair, he will need
to sell 400 sandwiches at $10 each. He relies on information about the number of
customers served by the average booth in previous years. The question asks
what most weakens Adam’s conclusion. The correct answer will either show that
Adam has overlooked other important costs, or that his logic is fundamentally flawed.
There could be more than one choice that seems to suggest that Adam
might not break even; the correct answer will be the one that eliminates any
possibility that he will break even.
(A) Though this is certainly true in a practical sense, it does not show that Adam's
plan will not work and that he will not stay within his budget. In fact, if he could
sell more than 400 sandwiches, it is likely that this would help him break even.
(B) CORRECT. Each sandwich costs some amount of money to make and sell,
so even if Adam sells the 400 sandwiches as planned, his net income will
certainly be less than $4,000. Based on this statement, Adam definitely will not
break even.
(C) Though it is true that an average number of customers means some booths
served more and some served fewer, this statement does not tell us that Adam
will have fewer than 400 or that his plan cannot work. Consider also that each
customer might buy more than one sandwich, so Adam could have fewer
customers but still sell enough sandwiches to cover his costs.
(D) Although some people will buy other types of food at the fair, this statement
fails to conclusively weaken Adam’s logic. Adam relied on information about the
average number of customers for food booths in previous years, but those
booths may have sold sandwiches or other foods – we simply don’t have enough
information to judge whether this statement strengthens or weakens Adam’s
conclusion.
(E) Sharing the booth would actually make it more likely that Adam would break
even. If he shares the cost of the booth rental, then he is more likely to sell
enough sandwiches to cover his costs.
7.
The argument concerns the economic impact on restaurants in Prohibitionland if
the service of alcoholic beverages is banned. It presents evidence that, despite
restrictions on the service of alcohol in certain areas of Prohibitionland, sales
taxes in restaurants in those areas rose at a higher rate than for those in other
parts of Prohibitionland, suggesting that the ban would not have any adverse
economic impact. We are asked to support the restaurant proprietors' claim , so
the correct answer choice will call the relevance of the seemingly contradictory
evidence into question.
(A). This answer choice may seem to strengthen the argument that banning the
service of alcoholic beverages would have an adverse impact on restaurants.
However, as the evidence involves data for the entire year, citing a short-term
negative impact on restaurant visitation at the beginning of the year does not
measurably strengthen the argument.
(B) The relative tax rate on food and beverages as compared to other consumer
good is irrelevant here.
(C) A gradual decline in alcohol consumption over the past 20 years would
suggest that over time, any ban on alcohol would have an increasingly small
impact on restaurant visitation, weakening the proprietors’ argument.
(D) CORRECT. This statement calls the evidence into question by indicating that
any measured increase in sales taxes and, presumably, revenues for restaurants
that have been operating under the restrictions last year enacted is irrelevant, as
the restrictions could be argued to be completely different than the total ban that
is being proposed. This answer choice substantially strengthens the proprietors’
argument by threatening to make the cited evidence irrelevant.
(E) The fact that overall sales tax revenue did not increase at a higher rate in the
provinces that enacted the restrictions on alcoholic beverages weakens the
proprietors’ argument, as it makes the cited evidence more compelling by ruling
out the possibility of different growth rates in the different areas.
8.
The official's conclusion is that people who claim that the U.S. is more vulnerable
than other nations because of the country's lack of a national vaccine laboratory
are disloyal and incorrect. His basis for that conclusion is that the U.S. has
generally long life span and low infant mortality relative to all UN countries.
Moreover, he cites the high quality of American hospitals, and he adds that many
people he knows from around the world come to the U.S. for medical care. We
are asked to find the choice that does NOT point out a weakness or potential
weakness in the official's argument.
(A) CORRECT. This choice states that the high quality of hospitals is not a factor
affecting the public's vulnerability to infectious disease. However, the quality of
hospitals very arguably does affect how vulnerable the public is to such disease.
(B) This choice highlights the official's logical jump from "disloyal" to "wrong" in
the phrase "these critics are disloyal and thus wrong about the public's
vulnerability." There is no necessary connection between disloyalty and
wrongness.
(C) If the Europeans that the official cited overwhelmingly consist of wealthy men
over the age of fifty, then the official relied on an unrepresentative sample to
justify his claim. What is true of wealthy older European men is not necessarily
true of Europeans or non-Americans generally.
(D) If the average life span of Americans is determined by causes other than
infectious disease, then the official is not limiting his evidence to cases relating to
vaccines. Other causes of death are not relevant to the critics' argument.
(E) The ranking of the United States relative to all UN countries is misleading,
since it does not compare the U.S. to other "advanced industrialized" or
"developed" nations, as the critics' claim does.
9.
The conclusion of the argument is that insurance companies do not have a
significant economic incentive to delay claim payments to doctors. To weaken
this conclusion, an answer choice must provide some significant economic
incentive for insurance companies to be tardy in paying doctors for legitimate
medical claims.
(A) While the fact that some doctors who submit accurate bills to insurance
companies still receive tardy payments seems to indicate that there must be
something other than errors causing delayed payments, it fails to prove that the
insurance company has an economic incentive to deliberately delay claim
payments to doctors. For example, this fact could simply indicate that the
insurance companies are inefficient at handling all of their paperwork.
(B) This choice compares costs insurance companies must absorb due to
incorrect bills to costs physicians must absorb due to tardy payments. However,
this information is irrelevant to establishing an economic incentive for insurance
companies to delay claim payments to doctors.
(C) The argument is focused on the payment of legitimate claims; the rising
proportion of illegitimate claims does not establish a clear economic incentive for
insurance companies to delay payments of legitimate claims.
(D) The types of billing errors made by doctors' offices does not establish any
economic motive for insurance companies to make a practice of delaying
payments to doctors.
(E) CORRECT. This choice articulates a logical chain that establishes a clear
economic motive for insurance companies to be tardy in paying doctors for
legitimate medical claims. If insurance companies delay payments to doctors,
this results in a 10 percent increase in overhead costs for physicians. These
costs ultimately result in higher fees that doctors charge to insurance companies.
Insurance companies, in turn, raise the premiums they charge consumers for
health coverage. This choice states that the insurance companies increase their
fees to consumers far more than the doctors increase their fees to insurance
companies, enabling the insurance companies to pocket the difference; therein
lies the economic motive for insurance companies to be tardy in paying doctors
for legitimate medical claims. 10.
This argument concludes that a decline in the percentage of retirees who
relocate to SunState will have a negative impact on businesses there that cater
to retirees. However, a decline in this percentage would only have a negative
impact on businesses if it indicated a decrease in the actual number of retirees. If
the actual number of retirees is steady or increasing, then a decrease in the
percentage wouldn't matter. As we are looking for a statement that weakens the
argument, we should look for an answer choice that somehow mitigates the
effect of this percentage decrease.
(A) The fact that SunState attracts more retirees than any other state does not
address the impact of the declining proportion of retirees moving to SunState.
(B) The existence of other businesses in SunState that do not cater to retirees is
not relevant.
(C) Any increase in departure of retirees from SunState to accept re-employment
would further damage businesses that serve retirees. However, the argument
explicitly discusses the impact of the declining percentage of retirees relocating
to SunState, and no other factors, making this answer choice irrelevant. In any
case, this answer choice suggests that such businesses will indeed lose
business, which would strengthen the conclusion, not weaken it.
(D) Low property taxes provide one reason why SunState is an appealing
destination for retirees, but this is not relevant in determining the economic
impact of the smaller proportion of retirees moving to SunState overall.
(E) CORRECT. If the total number of retirees that relocated to other states
increased significantly, a 10 percent reduction in the proportion of retirees that
moved to SunState may not result in a reduction in the actual number of people
who moved to SunState. This choice weakens the contention that businesses
that cater to retirees in SunState will suffer from a drop-off resulting from the
percentage decrease.
Alum
Joined: 19 Mar 2012
Posts: 4341
Own Kudos [?]: 51447 [19]
Given Kudos: 2326
Location: United States (WA)
Concentration: Leadership, General Management
Schools: Ross '20 (M)
GMAT 1: 760 Q50 V42
GMAT 2: 740 Q49 V42 (Online)
GMAT 3: 760 Q50 V42 (Online)
GPA: 3.8
WE:Marketing (Non-Profit and Government)
Send PM
Re: WEEK 1: CRITICAL REASONING STRENGTHEN/WEAKEN Questions [#permalink]
11
Kudos
8
Bookmarks
Expert Reply
methevoid wrote:
souvik101990 wrote:
[textarea]

WEEK 1: Strengthen/Weaken/Evaluate the Argument


STRATEGY : Strengthen Question




In the Strengthen Set I got 4 Wrongs - #1,3,7 and 8 out of 9, I had seen 1 question earlier so not counting it.

Took 21 Minutes to did these 9 questions.


In # 1 and # 7 I was totally bowled over, could not get understand the argument properly.

I feel #3 was a sitter, I could not have missed that.

Souvik, any suggestions on how to improve on Strengthen questions.


2 Basic approaches for a CR question



Inference/Bold Face question



In these type of question you should trust your core logic skills. Premises should be your best friend, and you should look out for excruciating details to deconstruct the argument. The quantitative part of your brain needs to be on high alert in this type of questions.

For example, what do you really look for an answer to an inference question?

Statements that are "Must be true" types, OR
Statements that can be logically proven to be true.

In these questions you need to focus on the flow of the argument, define roles for the premises and compare the answer choice. For these type of questions, paraphrasing is NOT a good idea.

Strengthen/weaken type of questions



These are just the opposite. In these types of questions, you need to look at the bigger picture. You can let your logic rest somewhere else, and let your common sense on the driver's seat. Paraphrasing is a highly advised strategy here.
Well look at it this way: we all did strengthening/weakening type of questions as a kid unknowingly.
See whether you are familiar with this situation.

You break the window glass of your neighbor's house while playing ball. He comes to your place and gives your dad a piece of his mind. Then you tell them that it wasn't you, and probably the other kids of the next block broke the glass.

See you weaken your neighbor's conclusion without even touching powerscore or manhattan.
That is why paraphrasing becomes easier, because you are already familiar with it.

My advice is to paraphrase all strenghten weaken questions without looking at the answer choice. Even if the option did not align with the OA, don't panic. It will, at least make you think in the right direction and let your brain know how to function on those questions when you take the gmat.

Hope this helps.

Souvik
Alum
Joined: 19 Mar 2012
Posts: 4341
Own Kudos [?]: 51447 [7]
Given Kudos: 2326
Location: United States (WA)
Concentration: Leadership, General Management
Schools: Ross '20 (M)
GMAT 1: 760 Q50 V42
GMAT 2: 740 Q49 V42 (Online)
GMAT 3: 760 Q50 V42 (Online)
GPA: 3.8
WE:Marketing (Non-Profit and Government)
Send PM
Re: GMATCLUB VERBAL ATTACK [#permalink]
6
Kudos
1
Bookmarks
Expert Reply

"ASSUMPTION" Set OE



1. The Police Commissioner's proposal hopes to decrease the number of crimes in
city Y by shifting police officers from low-crime to high-crime districts. His
proposal is based on data that demonstrate that crime decreases when
additional police officers are moved into a district. However, the data do not
mention anything about the effect on the districts from which the police officers
were removed. The commissioner's plan is based on the assumption that the
movement of police officers will not have any adverse effects on the low-crime
districts.
(A) While it is encouraging that a similar plan worked successfully in City X, this
fact is certainly not essential for the success of the plan in City Y. The cities may
be so different as to make the comparison meaningless.
(B) The police commissioner's proposal is focused solely on decreasing the
number of crimes in city Y. The severity of the crimes has no bearing on whether
the commissioner's proposal will succeed or not.
(C) The actual numerical distinction between high and low-crime areas of the city
is immaterial to the commissioner's proposal. For instance, if the number of
crimes committed in all high crime districts was only double (instead of more than
triple) the number of crimes committed in low crime districts, the proposal could
still be valid.
(D) It would be practically beneficial to the commissioner's plan if there were
more low crime than high crime districts in city Y. This would enable the
movement of police officers to every high crime district. However, this is not
necessary to achieve the commissioner's goal of decreasing the total number of
crimes in city Y. Even if there were more high-crime districts than low-crime
districts in city Y, police officers could still be shifted to some (though not all)
high-crime districts, and thereby possibly reduce the total number of crimes in
city Y.
(E) CORRECT. The police commissioner's proposal would not make sense if
districts of the city from which police officers are removed experience significant
crime increases shortly after the removal of those officers. This would at least
partially, if not fully, negate the reduction in the number of crimes in the highcrime districts.
This choice establishes that, in fact, the low-crime districts do
NOT suffer from significant crime increases after the removal of some officers--
an essential assumption upon which the commissioner's proposal depends.

2. The argument concludes that rising sea levels caused by global warming will
destroy major coastal population centers and displace millions of people. Any
assumption in support of this conclusion would have to corroborate that these
events will definitively take place.
(A) CORRECT. If new technological developments in the next century allow
people to divert rising seas from the world’s cities (i.e., population centers), cities
will not be destroyed and millions of people will not be displaced. Thus, a
necessary assumption is that these technologies will not be developed.
(B) A simple awareness of the steps to reduce emissions in no way undermines
the argument’s conclusion, as this answer choice does not describe any action
being taken by individuals. Additionally, greenhouse gases are never mentioned
as the primary by-product of human activity that causes global warming, and are
therefore not sufficient to address the argument.
(C) The argument never suggests that all coastal population centers are similarly
affected; this choice is too extreme and overreaching for the argument’s
conclusion.
(D) This might be true, but it is not an assumption on which the conclusion rests.
Instead, this answer choice is simply an inference that might be drawn from the
premises.
(E) The idea that human activity is the sole cause of global warming is neither
suggested nor assumed by the argument. In addition, the wording "sole cause" is
too extreme.

3. As an advertisement, this passage attempts to entice the reader into purchasing
a new HitItFar driver by touting its benefits, both implied or explicit. The key to
answering this question is to be able to analyze each claim to determine whether
it is implied, explicitly stated, or neither.
(A) By asking the reader rhetorically "isn't it time you added power ... and
distance ... [by switching to the HitItFar driver]?", the advertisement implies that
the use of the club will add "power ... and distance" and, hence, will improve
one's play.
(B) CORRECT. The advertisement states that the 12 major championships
winner have recently switched to the new driver. There is nothing in the passage
to imply that any of them were using the driver at the time of their victories;
hence, this claim is neither implied, nor made explicitly, in the passage. (C) It is a
reasonable assumption that professional golfers, particularly those
skilled enough to win a major championship, are experts and know what
constitutes a great club. The advertisement makes this implication and reinforces
it by citing the recent decision of these golfers to switch to the HitItFar driver; this
provides an implicit expert endorsement for the HitItFar driver.
(D) The point of the advertisement is to prompt the reader into purchasing a new
driver to replace his or her old driver in order to "add power and distance"; this
implies that the new driver is superior to the reader's existing driver.
(E) This claim is explicitly stated in the first sentence of the passage.

4. The researchers claim that Delta-32 prevents its carriers from contracting the
Plague. They support this claim by noting that a strikingly large percentage of
descendants of Plague survivors carry the mutation. We are asked to find an
assumption underlying the claim.
(A) The argument is specific to the relationship between Delta-32 and resistance
to the Plague. Other diseases are irrelevant.
(B) Again, the argument is specific to the relationship between Delta-32 and
resistance to the Plague. Other diseases are irrelevant.
(C) Delta-32 may have existed in its current form before the sixteenth century
and the merit of the argument would not change.
(D) The argument does not claim that Delta-32 prevents all bacteria-caused
disease.
(E) CORRECT. The researchers claim that Delta-32 prevented its carriers from
contracting the Plague on the basis of its presence in descendants of Plague
survivors. But it is theoretically possible that these descendants carry the
mutation Delta-32 because the Plague mutated the genes of their ancestors. In
order to claim that the mutation prevented the Plague, we must assume that the
Plague did not cause the mutation Delta-32.

5. The author concludes that one will only be able to determine the age of a
Brazilian ash by counting its rings if the temperature in the tree's environment
never exceeds 95 degrees Fahrenheit. The author bases this conclusion on the
fact that the tree loses rings when the temperature exceeds that level. However,
if the number of rings lost by a Brazilian ash at high temperatures can be
predicted, it may be possible to determine the age of a tree even if the
temperature exceeds 95 degrees. (A) The argument says nothing about precipitation.
This answer choice is out of scope since it would require a number of other assumptions
to make it relevant to the argument's conclusion.
(B) Whether other trees share this feature is irrelevant; the argument focuses
only on the Brazilian ash.
(C) The number of days of excessive heat needed to cause the tree to lose rings
is irrelevant.
(D) The thickness of the rings is irrelevant.
(E) CORRECT. The conclusion is that the rings will be a reliable measure only if
the temperature never exceeds 95 degrees. This is true only if there is no way to
predict how many rings would be lost when the temperature does exceed 95
degrees. (If it were possible to predict this, one might be able to assess the age
of a tree using its rings even if the temperature had exceeded 95 degrees.)

6. The conclusion of this argument is that the national identification system (“using
licenses for purposes not directly related to operating a motor vehicle”) is unAmerican.
The basis for this claim is that such a system would allow the
government to restrict the liberty of its people. The necessary assumption is one
that connects restrictions on liberties to the concept of “un-American” policies.
(A) The author never mentions future presidential elections, or the role of the
president in such a national identification system. Therefore, the conclusion that
the national identification system is un-American does not depend on this
assumption.
(B) Whether the government will soon, or will ever, start curtailing the activities of
dissidents is irrelevant to this argument: that the national identification system is
un-American simply because it restricts the liberties of U.S. citizens. Even if the
government does not abuse the power the national identification system
provides, the system could still be considered un-American.
(C) CORRECT. This choice connects the concept of "un-American" policies to
restrictions on liberties, essentially defining blanket restrictions on citizens as unAmerican.
(D) Whether Americans are willing to give up their right to travel freely is
irrelevant to this argument: that the national identification system is un-American
simply because it restricts the liberties of U.S. citizens. Even if Americans were willing to
give up their right to move about without identification, the system could
still be considered un-American.
(E) While the author may be inclined to agree that Americans should resist the
government regulation of their lives that the national identification system
represents, this argument does not depend on such an assumption. In fact, the
author makes a distinction between the national identification system and
“licenses for purposes…directly related to operating a motor vehicle,” so it is
possible that the author considers some government regulation reasonable.

7. The argument presents the facts of an apparent change in a magazine's cover
features since the new publisher took control. While a gossip columnist hailed the
change, newspaper editorials disagreed and concluded that the publisher
favored profit over reporting. The editorials are the opponents of the gossip
columnist; since their conclusion is about the publisher’s desires, there must be
an assumption connecting the publisher to the covers.
(A) This choice is irrelevant, as it is not connected to the conclusion. The
activities of celebrities have nothing to do with the publisher’s interests.
(B) CORRECT. Since the conclusion concerns the publisher’s desires based on
the content of the magazine covers, the editorials have to assume that the
publisher decides who is to be a cover subject. If not, there is no connection
between the covers and the publisher’s interests.
(C) This choice is the opposite of a necessary assumption. For the editorials to
conclude that the publisher prefers profits to reporting, they have to assume that
the two are mutually exclusive.
(D) “Some” means “at least one,” so this is not a powerful statement in any
direction. Furthermore, even if several such stars were running for political office,
it is not at all necessary to assume that to conclude that the publisher was more
interested in profits.
(E) This choice is not correct. While it is true that the editorials must assume
model and movie star covers are likely to sell more copies, it does not have to be
assumed that such covers will result in the sale of triple the number of copies, or
any other specific number.

8. The conclusion of the argument is that renewable sources of energy, chiefly solar
and wind, will be less risky for certain utilities than nonrenewable sources, such
as oil and gas. The basis for this claim is that the renewable sources will provide
stable, low-cost supplies of energy, whereas the prices for nonrenewable sources
will fluctuate according to availability. We are asked to find an
assumption underlying this argument. In order for this argument to be valid, it
must in fact be true that these renewable sources of energy will provide stable,
low-cost supplies.
(A) The utility companies' claim has to do with the supply risk of the new energy
sources, not with how these sources are received by the public.
(B) If no new supplies of traditional energy sources are found, then it is true that
perhaps these nonrenewable supplies will continue to fluctuate in price in a risky
manner. However, the argument does not depend upon any assumption about
the future discovery of oil and gas supplies.
(C) CORRECT. If we assume that weather patterns are consistent and
predictable, then with the stated premises, we can conclude that solar and wind
power will be less risky than oil and gas. If, on the other hand, weather patterns
are not consistent and predictable, then solar and wind power are not reliable
and thus will not provide "stable energy supplies at low cost." Thus, the
argument's conclusion directly depends on this assumption.
(D) To reach the required conclusion, it is not necessary to assume that the
conversion technology for new sources is not more expensive than the present
technology.
(E) This choice does not directly affect the argument. Whether or not energy
produced through combustion can be made less risky, the new energy sources
might still be less risky than the older sources.

9. We are given two premises based on survey results: first, vanilla is the bestselling
flavor of ice cream and, second, those who prefer chocolate usually don't
order vanilla. The author concludes that vanilla-flavored candy should sell better
than chocolate-flavored candy. The author bases this conclusion on the
assumption that it is valid to extend the survey's results beyond ice cream to
include candy.
(A) As a stand-alone, this choice makes common sense but, in the argument, it
would undermine the author's conclusion. We are asked to find an assumption
upon which the author relies, which means the correct assumption should
support the author's conclusion.
(B) This choice is either irrelevant at best (the survey does not address children
specifically) or would undermine the author's conclusion, at worst. We are asked
to find an assumption upon which the author relies, which means the correct
assumption should support the author's conclusion. (C) Preferences for flavors
neither vanilla nor chocolate are outside of the scope of this argument.
(D) This choice addresses only ice cream preferences; it does not provide any
information to tie ice cream preferences to candy preferences.
(E) CORRECT. This assumption supports the author's conclusion by tying ice
cream preferences directly to candy preferences.

10. The conclusion of the argument is that the media are wrong in saying that the
economy is entering a phase of growth and prosperity. The basis for that claim is
that the number of people filing for bankruptcy has increased every month for the
last six months and that bankruptcy lawyers are busier than they have been in
years. In order for this argument to be valid, however, the author has to assume
that the increase in the number of bankruptcies is a result of the state of the
economy and not the result of something unrelated.
(A) This statement does not have to be true for the claim that the media are
wrong about the economy to hold. Even if unemployment rates are useful
indicators of growth and prosperity, the media could still be wrong about the
economy (e.g., if there are other indicators that show problems in other areas).
(B) This does not have to be true for the conclusion to hold. Productivity could
be a good measure of economic growth, but the media could still be wrong about
the economy (e.g., if there are other indicators that show problems in other
areas).
(C) CORRECT. This has to be true for the conclusion to hold. If legislation has
recently been passed that makes it easier to obtain bankruptcy, then the
evidence cited would be less relevant. The increased number of bankruptcies
could have been the result of the easier process rather than of a poor economy.
(D) This does not have to be true for the conclusion to hold. An increase in the
number of bankruptcy lawyers would not explain the increase in the number of
bankruptcy filings.
(E) This does not have to be true for the claim that the media are wrong about
the economy to hold. Even if the media did not often misrepresent the current
state of economic affairs, the argument that the media are wrong might still hold.
Alum
Joined: 19 Mar 2012
Posts: 4341
Own Kudos [?]: 51447 [2]
Given Kudos: 2326
Location: United States (WA)
Concentration: Leadership, General Management
Schools: Ross '20 (M)
GMAT 1: 760 Q50 V42
GMAT 2: 740 Q49 V42 (Online)
GMAT 3: 760 Q50 V42 (Online)
GPA: 3.8
WE:Marketing (Non-Profit and Government)
Send PM
I received a PM regarding this [#permalink]
2
Kudos
Expert Reply
Q. Since the passage of the state’s Clean Air Act ten years ago, the level of industrial pollutants in the air has fallen by an average of 18 percent. This suggests that the restrictions on industry embodied in the act have worked effectively. However, during the same period the state has also suffered through a period of economic decline. The number of businesses in the state has fallen by 10 percent, and the number of workers employed has fallen by 12 percent. It is probable that the business decline, rather than the regulations in the act, is responsible for at least half of the decline in the pollution.

Which of the following, if true, would most seriously weaken the conclusion drawn in the passage above?

(A) During the last ten years, economic conditions in the nation as a whole have been worse than those within the state.
(B) Amendments to the Clean Air Act that were enacted six years ago have substantially strengthened its restrictions on industrial air pollution.
(C) Of the businesses that ceased operating in the state during the last ten years, only 5 percent were engaged in air-polluting industries.
(D) Several large corporations left the state during the last ten years partly in order to avoid compliance with the Clean Air Act.
(E) Due to its small budget, the state office charged with enforcement of the Clean Air Act has prosecuted only two violators of the law since its passage.

CONCLUSION: It is probable that the business decline, rather than the regulations in the act, is responsible for at least half of the decline in the pollution.

FACTS: Pollutants down by 18%, business down by 10% and workers down by 12%

Prephrasing: We need something that gives us a signal that the ACT and not the business decline is responsible for the decline

A) doesnt address any of the 2 issues
B) this is weird. Why would stricter regulations be responsible? Also this does not addresses the business decline
D) A lot of businesses left so it appears that pollution was less because of business withdrawal. Strengthens.
E) Says that the the ACT has not been very helpful. Strengthens again.


C) This means that the major businesses that cause pollution are still there. So it must be the ACT that has reduced the pollution. Good One. CORRECT!
Alum
Joined: 19 Mar 2012
Posts: 4341
Own Kudos [?]: 51447 [4]
Given Kudos: 2326
Location: United States (WA)
Concentration: Leadership, General Management
Schools: Ross '20 (M)
GMAT 1: 760 Q50 V42
GMAT 2: 740 Q49 V42 (Online)
GMAT 3: 760 Q50 V42 (Online)
GPA: 3.8
WE:Marketing (Non-Profit and Government)
Send PM
Re: GMATCLUB VERBAL ATTACK [#permalink]
1
Kudos
3
Bookmarks
Expert Reply

SENTENCE CORRECTION ATTACK: MODIFIERS!



No theory! We will learn with the questions and answers!!


1. Although covered in about 11 inches of snow, aviation officials said that conditions on the
runway at the time of the emergency landing was acceptable.
• aviation officials said that conditions on the runway at the time of the emergency landing was
acceptable
• the runway conditions during the emergency landing were acceptable according to aviation
officials
• according to aviation officials, the runway was in acceptable condition during the time of the
emergency landing
• the runway was in acceptable condition during the emergency landing, according to aviation
officials
• aviation officials said that conditions on the runway at the time of the emergency landing were
acceptable

2. Discouraged by new data that show increases in toxic emissions from domestic factories,
searches for alternative investment opportunities are being conducted by shareholders of the
nation’s leading manufacturing companies.
• searches for alternative investment opportunities are being conducted by shareholders of the
nation’s leading manufacturing companies
• searches are being conducted by shareholders of the nation’s leading manufacturing
companies who are looking for alternative investment opportunities
• shareholders of the nation’s leading manufacturing companies had begun searching for
investment opportunities outside of the manufacturing industry
• the nation’s leading manufacturing companies are searching for alternative investment
opportunities for its shareholders
• shareholders of the nation’s leading manufacturing companies are searching for alternative
investment opportunities

3. Found in the wild only in Australia and New Guinea, powerful back legs with long feet
distinguish kangaroos from other large mammals.
• powerful back legs with long feet distinguish kangaroos from other large mammals
• powerful back legs with long feet distinguish kangaroos from other mammals that are large
• powerful back legs with long feet distinguish kangaroos from those of other mammals that are
large
• kangaroos are distinguished from other large mammals by powerful legs with long feet
• kangaroos are being distinguished from other mammals that are large by powerful legs with
long feet
4. Responding to growing demand for high-end vehicles, the interiors of the newest models are
so luxurious that they sell for nearly twice the price of last year's models.
• the interiors of the newest models are so luxurious that they sell
• the interiors of the newest models are so luxurious that the cars are sold
• auto makers have installed interiors in the newest models that are so luxurious that they sell
• the interior of the newest models are so luxurious that they are sold
• auto makers have installed such luxurious interiors in the newest models that these cars sell

5. By applying optimization techniques commonly used to plan operations, it is possible to
determine how much effort ought to be devoted to each of a company’s products in order to
meet its goals in both the short and long terms.
• it is possible to determine how much effort ought to be devoted to each of a company’s
products in order to meet its goals in both the short and long terms
• a company’s managers can determine how much effort should be dedicated to each of the
company’s products in order to meet its short and long term goals
• it can be determined by company managers how much effort ought to be devoted to each of
the company’s products in order to meet its goals, both short and long term
• it may be possible for company managers to determine how much effort should be dedicated
to each of these products in order to meet the company’s short and long term goals
• managers at a company can determine how much effort ought to be dedicated to each of
these products in order to meet the company’s goals in both the short and long term

6. Given its authoritative coverage of other science topics, the textbook's chapter on genetics is
surprisingly tentative, which leads one to doubt the author's scholarship in that particular area.
• the textbook's chapter on genetics is surprisingly tentative, which leads
• the chapter of the textbook on genetics is surprisingly tentative, leading
• the textbook contains a surprising and tentative chapter on genetics, which leads
• the textbook's chapter on genetics is surprisingly tentative and leads
• the textbook is surprisingly tentative in its chapter on genetics, leading

7. Hailed as a key discovery in the science of evolution, the fossils of a large scaly creature
resembling both a fish and a land-animal provide evidence of a possible link in the
evolutionary chain from water-based to land-based organisms.
• the fossils of a large scaly creature resembling both a fish and a land-animal provide evidence
of
• a large scaly creature resembling both a fish and a land-animal provides fossils that are
evidence
• a large scaly creature, whose fossils resemble both a fish and a land-animal, provides evidence
of
• the fossils of a large scaly creature, which resembles both a fish and a land-animal, provides
evidence of
• the fossils of a large scaly creature resemble both a fish and a land-animal and provide
evidence of

8. Hoping to alleviate some of the financial burdens of a growing population, property taxes last
year were raised by an eleven percent increase by the county government.
• property taxes last year were raised by an eleven percent increase by the county government
• property taxes were raised by eleven percent last year by the county government
• the county government raised property taxes by an eleven percent increase last year
• the county government last year raised by eleven percent property taxes
• the county government raised property taxes by eleven percent last year

9. In order to properly evaluate a patient’s state of mind and gain informed consent prior to
surgery, a substantial period of time must be spent with the operating physician by the patient
to become fully aware of the pros and cons of undergoing a surgical procedure.
• a substantial period of time must be spent with the operating physician by the patient to
become fully aware of the pros and cons of undergoing a surgical procedure
• the operating physician and the patient must spend a substantial amount of time together,
thus ensuring full awareness of the pros and cons of undergoing the surgical procedure
• the patient must spend a substantial amount of time with his or her operating physician, thus
ensuring that he or she has been made fully aware of the pros and cons of undergoing the
surgical procedure
• the operating physician must spend a substantial amount of time with the patient, thus
ensuring that the patient is fully aware of the pros and cons of accepting the undergoing
procedure
• the operating physician must ensure that he or she is fully aware of the pros and cons of
undergoing a surgical procedure by spending a substantial amount of time with the patient

10. Many daring vacationers who participate in guided boat tours on the Tarcoles River encounter
native crocodiles lurking in the shallows, whose eyes and noses are peaking out from the
surface of the murky water.
• encounter native crocodiles lurking in the shallows,whose eyes and noses are peaking out
• encountered native crocodiles lurking in the shallows, whose eyes and noses peak out
• had encountered native crocodiles lurking in the shallows, whose eyes and noses peak out
• encounter native crocodiles lurking in the shallows,with eyes and noses peaking out
• encounter native crocodiles lurking in the shallows,with eyes and noses that are peaking out

11. Before its independence in 1947, Britain ruled India as a colony and they would relinquish
power only after a long struggle by the native people.
• Before its independence in 1947, Britain ruled India as a colony and they would relinquish
power
• Before independence in 1947, Britain had ruled India as a colony and relinquished power
• Before its independence in 1947, India was ruled by Britain as a colony and they relinquished
power
• Before independence in 1947, India had been ruled as a colony by Britain, which relinquished
power
• Before independence in 1947, India had been a colony of the British, who relinquished power
12. Used until the end of the Second World War, the German army employed the U-boat to attack
both military or civilian watercraft.
• the German army employed the U-boat to attack both military or
• the U-boat was employed by the German army to attack both military and
• the U-boat employed the German army to attack both military or
• the German army had employed the U-Boat to attack both military and the
• the U-boat has been employed by the German army to attack both military and also

13. Though most people take it for granted now, the nation wide admission of students to colleges
and universities based on academic merit is a relatively recent phenomenon, beginning only
after World War II.
• Though most people take it for granted now, the nationwide admission of students to colleges
and universities based on academic merit
• Though it is now taken for granted by most people, the admission of nationwide students to
colleges and universities based on academic merit
• Now taken for granted by most people, colleges and universities admitting students based on
their academic merit
• Most take them for granted now, but the admission of nationwide students to colleges and
universities based on their academic merit
• Most people now take for granted that colleges and universities admit students nationally
based on academic merit, and it

14. According to Italy's top anti-Mafia prosecutor, the ailing mobster came to take refuge in
Corleone, a town famous because of the “The Godfather” and near to those he most trusted.
• the ailing mobster came to take refuge in Corleone,a town famous because of “The
Godfather” and near to those he most trusted
• famous because of “The Godfather,” the ailing mobster came to take refuge in Corleone, a
town near to those he most trusted
• the ailing mobster, famous because of “The Godfather,” came to take refuge in Corleone, a
town near to those he most trusted
• near to those he most trusted, the ailing mobster came to take refuge in Corleone, a town
famous because of “The Godfather”
• Corleone, famous because of “The Godfather,” was thetown that the ailing mobster came to
take refuge in because it was near to those he most trusted

15. The author Herman Melville and the poet Walt Whitman are icons of American literature,
greatly beloved by generations past and present.
• The author Herman Melville and the poet Walt Whitman are icons
• Herman Melville the author and Walt Whitman the poet are icons
• The author named Herman Melville and the poet named Walt Whitman are great icons
• The author, Herman Melville, and the poet, Walt Whitman, are icons
• Herman Melville, the author, and Walt Whitman, the poet, had been icons

16. Jean-Jacques Rousseau contended that man is good only when in "the state of nature" but is
corrupted by society, that compels man to compare himself to others.
• man is good only when in "the state of nature" but is corrupted by society, that
• only man is good when in "the state of nature" but is corrupted by society, that
• man is good when in "the state of nature" but is corrupted only by society, that
• only man is good when in "the state of nature" but is corrupted by society, which
• man is good only when in "the state of nature" but is corrupted by society, which

17. Though the language of Beowulf is practically incomprehensible to contemporary readers,
careful linguistic analysis reveals a multitude of similarities to modern English.
• Though the language of Beowulf is practically incomprehensible to contemporary readers,
careful linguistic analysis reveals a multitude of similarities to modern English.
• Despite that it is practically incomprehensible to contemporary readers, careful linguistic
analysis reveals that the language of Beowulf has a multitude of similarities to modern English.
• Though being practically incomprehensible to contemporary readers, the language of Beowulf
reveals through careful linguistic analysis a multitude of similarities to modern English.
• Though Beowulf has a language that is practically incomprehensible to contemporary readers,
a multitude of similarities are revealed to modern English through careful linguistic analysis.
• Despite having practically incomprehensible language to contemporary readers, Beowulf
reveals through careful linguistic analysis a multitude of similarities to modern English.

18. Fusion, the process through which the sun produces heat and light, has been studied by
scientists,some of whom have attempted to mimic the process intheir laboratories by blasting
a container of liquid solvent with strong ultrasonic vibrations.
• Fusion, the process through which the sun produces heat and light, has been studied by
scientists,
• Fusion, the heat and light produced by the sun, has been studied by scientists,
• Fusion, the process through which heat and light are produced by the sun, has been studied
by scientists,
• Scientists have studied fusion, the process the sun uses to produce heat and light,
• Scientists have studied fusion, the process the sun uses to produce heat and light, and

19. Pests had destroyed grape, celery, chili pepper crops, sugar beet and walnut in the region, but
in the 1880s, more effective pest-control methods saved the citrus industry.
• Pests had destroyed grape, celery, chili pepper crops,sugar beet and walnut in the region, but
in the 1880s, more effective pest-control methods saved the citrus industry.
• Pests had destroyed grape, celery, chili pepper, sugar beet and walnut crops in the region, but
in the 1880s, more effective pest-control methods saved the citrus industry.
• Pests had destroyed grape, celery, chili pepper, sugar beet and walnut crops in the region, but
more effective pest-control methods that were introduced in the 1880s saved the citrus
industry.
• In the 1880s, pests destroyed grape, celery, chili pepper, sugar beet and walnut crops in the
region and more effective pest-control methods saved the citrus industry.
• In the 1880s, more effective pest-control methods saved the citrus industry from what was
destroying grape, celery, chili pepper, sugar beet and walnut crops in the region.

20. Classical guitar was neither prestigious nor was often played in concert halls until it was
revived by Andres Segovia in the mid-twentieth century, having been won over by the
instrument's sound despite its relative obscurity.
• Classical guitar was neither prestigious nor was often played in concert halls until it was
revived by Andres Segovia in the mid-twentieth century, having been won over by the
instrument's sound despite its relative obscurity.
• Classical guitar was neither prestigious nor played often in concert halls until it was revived by
Andres Segovia in the mid-twentieth century, having been won over by the instrument's sound
despite its relative obscurity.
• Classical guitar was not prestigious and was not often played in concert halls until Andres
Segovia revived it in the mid-twentieth century, after he was won over by the sound despite
the instrument's relative obscurity.
• Classical guitar did not have prestige nor was it performed often in concert halls until its
revival by Andres Segovia, who in the mid-twentieth century was won over by the instrument's
sound despite its relative obscurity.
• Classical guitar was neither prestigious nor was often played in concert halls until Andres
Segovia revived it in the mid-twentieth century, when he was won over by the sound of the
relatively obscure instrument.

21. The physicist Richard Feynman presented a comprehensive introduction to modern physics
designed for undergraduate students in a two-year course.
• The physicist Richard Feynman presented a comprehensive introduction to modern physics
designed for undergraduate students in a two-year course.
• For undergraduate students, the physicist Richard Feynman presented a two-year course,
being a comprehensive introduction to modern physics.
• A comprehensive introduction was in a two-year course by the physicist Richard Feynman
presenting to undergraduate students an introduction to modern physics.
• Presenting a comprehensive introduction, the physicist Richard Feynman introduced modern
physics in a two-year course designed for undergraduate students.
• In a two-year course designed for undergraduate students, the physicist Richard Feynman
presented a comprehensive introduction to modern physics.

22. Descending approximately 4,000 years ago from the African wildcat, it has been an
exceedingly short time for the domestic cat with respect to genetic evolution and it scarcely
seems sufficient to allow the marked physical changes that transformed the animal.
• Descending approximately 4,000 years ago from the African wildcat, it has been an
exceedingly short time for the domestic cat with respect to genetic evolution and it scarcely
seems sufficient to allow the marked physical changes that transformed the animal.
• The domestic cat descended from the African wildcat approximately 4,000 years ago, which is
an exceedingly short time for the domestic cat's genetic evolution and scarcely sufficient for
the marked physical changes that transformed the animal.
• Descending from the African wildcat approximately 4,000 years ago, the domestic cat has had
an exceedingly short time for its genetic evolution and has been scarcely sufficient for the
marked physical changes in the animal.
• Having descended from the African wildcat approximately 4,000 years ago, the domestic cat
has had an exceedingly short time for its genetic evolution that has scarcely been sufficient for
the marked physical changes that transformed the animal.
• The domestic cat descended from the African wildcat approximately 4,000 years ago, an
exceedingly recent divergence with respect to genetic evolution and one which scarcely seems
sufficient to allow the marked physical changes in the animal.


Sorry about no underlining and a couple of Typos!
Otherwise, its a killer set!
Board of Directors
Joined: 01 Sep 2010
Posts: 4381
Own Kudos [?]: 32868 [4]
Given Kudos: 4453
Send PM
Re: GMATCLUB VERBAL ATTACK [#permalink]
4
Kudos
Quote:
1. Although covered in about 11 inches of snow, aviation officials said that conditions on the
runway at the time of the emergency landing was acceptable
.

• aviation officials said that conditions on the runway at the time of the emergency landing was
acceptable
• the runway conditions during the emergency landing were acceptable according to aviation
officials
• according to aviation officials, the runway was in acceptable condition during the time of the
emergency landing
• the runway was in acceptable condition during the emergency landing, according to aviation
officials
• aviation officials said that conditions on the runway at the time of the emergency landing were
acceptable

2. Discouraged by new data that show increases in toxic emissions from domestic factories,
searches for alternative investment opportunities are being conducted by shareholders of the
nation’s leading manufacturing companies.


• searches for alternative investment opportunities are being conducted by shareholders of the
nation’s leading manufacturing companies
• searches are being conducted by shareholders of the nation’s leading manufacturing
companies who are looking for alternative investment opportunities
• shareholders of the nation’s leading manufacturing companies had begun searching for
investment opportunities outside of the manufacturing industry
• the nation’s leading manufacturing companies are searching for alternative investment
opportunities for its shareholders
• shareholders of the nation’s leading manufacturing companies are searching for alternative
investment opportunities

3. Found in the wild only in Australia and New Guinea, powerful back legs with long feet
distinguish kangaroos from other large mammals.


• powerful back legs with long feet distinguish kangaroos from other large mammals
• powerful back legs with long feet distinguish kangaroos from other mammals that are large
• powerful back legs with long feet distinguish kangaroos from those of other mammals that are
large
• kangaroos are distinguished from other large mammals by powerful legs with long feet
• kangaroos are being distinguished from other mammals that are large by powerful legs with
long feet

4. Responding to growing demand for high-end vehicles, the interiors of the newest models are
so luxurious that they sell
for nearly twice the price of last year's models.

• the interiors of the newest models are so luxurious that they sell
• the interiors of the newest models are so luxurious that the cars are sold
• auto makers have installed interiors in the newest models that are so luxurious that they sell
• the interior of the newest models are so luxurious that they are sold
• auto makers have installed such luxurious interiors in the newest models that these cars sell

5. By applying optimization techniques commonly used to plan operations, it is possible to
determine how much effort ought to be devoted to each of a company’s products in order to
meet its goals in both the short and long terms.


• it is possible to determine how much effort ought to be devoted to each of a company’s
products in order to meet its goals in both the short and long terms
• a company’s managers can determine how much effort should be dedicated to each of the
company’s products in order to meet its short and long term goals
• it can be determined by company managers how much effort ought to be devoted to each of
the company’s products in order to meet its goals, both short and long term
• it may be possible for company managers to determine how much effort should be dedicated
to each of these products in order to meet the company’s short and long term goals
• managers at a company can determine how much effort ought to be dedicated to each of
these products in order to meet the company’s goals in both the short and long term

6. Given its authoritative coverage of other science topics, the textbook's chapter on genetics is
surprisingly tentative, which leads
one to doubt the author's scholarship in that particular area.

• the textbook's chapter on genetics is surprisingly tentative, which leads
• the chapter of the textbook on genetics is surprisingly tentative, leading
• the textbook contains a surprising and tentative chapter on genetics, which leads
• the textbook's chapter on genetics is surprisingly tentative and leads
• the textbook is surprisingly tentative in its chapter on genetics, leading

7. Hailed as a key discovery in the science of evolution, the fossils of a large scaly creature
resembling both a fish and a land-animal provide evidence of
a possible link in the
evolutionary chain from water-based to land-based organisms.

• the fossils of a large scaly creature resembling both a fish and a land-animal provide evidence
of
• a large scaly creature resembling both a fish and a land-animal provides fossils that are
evidence
• a large scaly creature, whose fossils resemble both a fish and a land-animal, provides evidence
of
• the fossils of a large scaly creature, which resembles both a fish and a land-animal, provides
evidence of
• the fossils of a large scaly creature resemble both a fish and a land-animal and provide
evidence of

8. Hoping to alleviate some of the financial burdens of a growing population, property taxes last
year were raised by an eleven percent increase by the county government.


• property taxes last year were raised by an eleven percent increase by the county government
• property taxes were raised by eleven percent last year by the county government
• the county government raised property taxes by an eleven percent increase last year
• the county government last year raised by eleven percent property taxes
• the county government raised property taxes by eleven percent last year

9. In order to properly evaluate a patient’s state of mind and gain informed consent prior to
surgery, a substantial period of time must be spent with the operating physician by the patient
to become fully aware of the pros and cons of undergoing a surgical procedure.


• a substantial period of time must be spent with the operating physician by the patient to
become fully aware of the pros and cons of undergoing a surgical procedure
• the operating physician and the patient must spend a substantial amount of time together,
thus ensuring full awareness of the pros and cons of undergoing the surgical procedure
• the patient must spend a substantial amount of time with his or her operating physician, thus
ensuring that he or she has been made fully aware of the pros and cons of undergoing the
surgical procedure
• the operating physician must spend a substantial amount of time with the patient, thus
ensuring that the patient is fully aware of the pros and cons of accepting the undergoing
procedure
• the operating physician must ensure that he or she is fully aware of the pros and cons of
undergoing a surgical procedure by spending a substantial amount of time with the patient

10. Many daring vacationers who participate in guided boat tours on the Tarcoles River encounter
native crocodiles lurking in the shallows, whose eyes and noses are peaking out
from the
surface of the murky water.

• encounter native crocodiles lurking in the shallows,whose eyes and noses are peaking out
• encountered native crocodiles lurking in the shallows, whose eyes and noses peak out
• had encountered native crocodiles lurking in the shallows, whose eyes and noses peak out
• encounter native crocodiles lurking in the shallows,with eyes and noses peaking out
• encounter native crocodiles lurking in the shallows,with eyes and noses that are peaking out

11. Before its independence in 1947, Britain ruled India as a colony and they would relinquish
power
only after a long struggle by the native people.

• Before its independence in 1947, Britain ruled India as a colony and they would relinquish
power
• Before independence in 1947, Britain had ruled India as a colony and relinquished power
• Before its independence in 1947, India was ruled by Britain as a colony and they relinquished
power
• Before independence in 1947, India had been ruled as a colony by Britain, which relinquished
power
• Before independence in 1947, India had been a colony of the British, who relinquished power

12. Used until the end of the Second World War, the German army employed the U-boat to attack
both military or
civilian watercraft.

• the German army employed the U-boat to attack both military or
• the U-boat was employed by the German army to attack both military and
• the U-boat employed the German army to attack both military or
• the German army had employed the U-Boat to attack both military and the
• the U-boat has been employed by the German army to attack both military and also

13. Though most people take it for granted now, the nation wide admission of students to colleges
and universities based on academic merit
is a relatively recent phenomenon, beginning only
after World War II.

• Though most people take it for granted now, the nationwide admission of students to colleges
and universities based on academic merit
• Though it is now taken for granted by most people, the admission of nationwide students to
colleges and universities based on academic merit
• Now taken for granted by most people, colleges and universities admitting students based on
their academic merit
• Most take them for granted now, but the admission of nationwide students to colleges and
universities based on their academic merit
• Most people now take for granted that colleges and universities admit students nationally
based on academic merit, and it

14. According to Italy's top anti-Mafia prosecutor, the ailing mobster came to take refuge in
Corleone, a town famous because of the “The Godfather” and near to those he most trusted.

• the ailing mobster came to take refuge in Corleone,a town famous because of “The
Godfather” and near to those he most trusted
• famous because of “The Godfather,” the ailing mobster came to take refuge in Corleone, a
town near to those he most trusted
• the ailing mobster, famous because of “The Godfather,” came to take refuge in Corleone, a
town near to those he most trusted
• near to those he most trusted, the ailing mobster came to take refuge in Corleone, a town
famous because of “The Godfather”
• Corleone, famous because of “The Godfather,” was thetown that the ailing mobster came to
take refuge in because it was near to those he most trusted

15. The author Herman Melville and the poet Walt Whitman are icons of American literature,
greatly beloved by generations past and present.

• The author Herman Melville and the poet Walt Whitman are icons
• Herman Melville the author and Walt Whitman the poet are icons
• The author named Herman Melville and the poet named Walt Whitman are great icons
• The author, Herman Melville, and the poet, Walt Whitman, are icons
• Herman Melville, the author, and Walt Whitman, the poet, had been icons

16. Jean-Jacques Rousseau contended that man is good only when in "the state of nature" but is
corrupted by society, that
compels man to compare himself to others.

• man is good only when in "the state of nature" but is corrupted by society, that
• only man is good when in "the state of nature" but is corrupted by society, that
• man is good when in "the state of nature" but is corrupted only by society, that
• only man is good when in "the state of nature" but is corrupted by society, which
• man is good only when in "the state of nature" but is corrupted by society, which

17. Though the language of Beowulf is practically incomprehensible to contemporary readers,
careful linguistic analysis reveals a multitude of similarities to modern English.

• Though the language of Beowulf is practically incomprehensible to contemporary readers,
careful linguistic analysis reveals a multitude of similarities to modern English.
• Despite that it is practically incomprehensible to contemporary readers, careful linguistic
analysis reveals that the language of Beowulf has a multitude of similarities to modern English.
• Though being practically incomprehensible to contemporary readers, the language of Beowulf
reveals through careful linguistic analysis a multitude of similarities to modern English.
• Though Beowulf has a language that is practically incomprehensible to contemporary readers,
a multitude of similarities are revealed to modern English through careful linguistic analysis.
• Despite having practically incomprehensible language to contemporary readers, Beowulf
reveals through careful linguistic analysis a multitude of similarities to modern English.

18. Fusion, the process through which the sun produces heat and light, has been studied by
scientists,
some of whom have attempted to mimic the process intheir laboratories by blasting
a container of liquid solvent with strong ultrasonic vibrations.

• Fusion, the process through which the sun produces heat and light, has been studied by
scientists,
• Fusion, the heat and light produced by the sun, has been studied by scientists,
• Fusion, the process through which heat and light are produced by the sun, has been studied
by scientists,
• Scientists have studied fusion, the process the sun uses to produce heat and light,
• Scientists have studied fusion, the process the sun uses to produce heat and light, and

19. Pests had destroyed grape, celery, chili pepper crops, sugar beet and walnut in the region, but
in the 1880s, more effective pest-control methods saved the citrus industry.


• Pests had destroyed grape, celery, chili pepper crops,sugar beet and walnut in the region, but
in the 1880s, more effective pest-control methods saved the citrus industry.
• Pests had destroyed grape, celery, chili pepper, sugar beet and walnut crops in the region, but
in the 1880s, more effective pest-control methods saved the citrus industry.
• Pests had destroyed grape, celery, chili pepper, sugar beet and walnut crops in the region, but
more effective pest-control methods that were introduced in the 1880s saved the citrus
industry.
• In the 1880s, pests destroyed grape, celery, chili pepper, sugar beet and walnut crops in the
region and more effective pest-control methods saved the citrus industry.
• In the 1880s, more effective pest-control methods saved the citrus industry from what was
destroying grape, celery, chili pepper, sugar beet and walnut crops in the region.

20. Classical guitar was neither prestigious nor was often played in concert halls until it was
revived by Andres Segovia in the mid-twentieth century, having been won over by the
instrument's sound despite its relative obscurity.


• Classical guitar was neither prestigious nor was often played in concert halls until it was
revived by Andres Segovia in the mid-twentieth century, having been won over by the
instrument's sound despite its relative obscurity.
• Classical guitar was neither prestigious nor played often in concert halls until it was revived by
Andres Segovia in the mid-twentieth century, having been won over by the instrument's sound
despite its relative obscurity.
• Classical guitar was not prestigious and was not often played in concert halls until Andres
Segovia revived it in the mid-twentieth century, after he was won over by the sound despite
the instrument's relative obscurity.
• Classical guitar did not have prestige nor was it performed often in concert halls until its
revival by Andres Segovia, who in the mid-twentieth century was won over by the instrument's
sound despite its relative obscurity.
• Classical guitar was neither prestigious nor was often played in concert halls until Andres
Segovia revived it in the mid-twentieth century, when he was won over by the sound of the
relatively obscure instrument.

21. The physicist Richard Feynman presented a comprehensive introduction to modern physics
designed for undergraduate students in a two-year course.

• The physicist Richard Feynman presented a comprehensive introduction to modern physics
designed for undergraduate students in a two-year course.
• For undergraduate students, the physicist Richard Feynman presented a two-year course,
being a comprehensive introduction to modern physics.
• A comprehensive introduction was in a two-year course by the physicist Richard Feynman
presenting to undergraduate students an introduction to modern physics.
• Presenting a comprehensive introduction, the physicist Richard Feynman introduced modern
physics in a two-year course designed for undergraduate students.
• In a two-year course designed for undergraduate students, the physicist Richard Feynman
presented a comprehensive introduction to modern physics.

22. Descending approximately 4,000 years ago from the African wildcat, it has been an
exceedingly short time for the domestic cat with respect to genetic evolution and it scarcely
seems sufficient to allow the marked physical changes that transformed the animal.


• Descending approximately 4,000 years ago from the African wildcat, it has been an
exceedingly short time for the domestic cat with respect to genetic evolution and it scarcely
seems sufficient to allow the marked physical changes that transformed the animal.
• The domestic cat descended from the African wildcat approximately 4,000 years ago, which is
an exceedingly short time for the domestic cat's genetic evolution and scarcely sufficient for
the marked physical changes that transformed the animal.
• Descending from the African wildcat approximately 4,000 years ago, the domestic cat has had
an exceedingly short time for its genetic evolution and has been scarcely sufficient for the
marked physical changes in the animal.
• Having descended from the African wildcat approximately 4,000 years ago, the domestic cat
has had an exceedingly short time for its genetic evolution that has scarcely been sufficient for
the marked physical changes that transformed the animal.
• The domestic cat descended from the African wildcat approximately 4,000 years ago, an
exceedingly recent divergence with respect to genetic evolution and one which scarcely seems
sufficient to allow the marked physical changes in the animal.


I did it for souvik :) enjoy
User avatar
Manager
Manager
Joined: 22 Dec 2011
Posts: 175
Own Kudos [?]: 1042 [0]
Given Kudos: 32
Send PM
Re: GMATCLUB VERBAL ATTACK [#permalink]
Hi Souvik - Do you have any set that comprises of Number and percentage CR? I guess they appear more often in all types of question. Any such set will be of great help......

Eagerly awaiting for your response....
Alum
Joined: 19 Mar 2012
Posts: 4341
Own Kudos [?]: 51447 [0]
Given Kudos: 2326
Location: United States (WA)
Concentration: Leadership, General Management
Schools: Ross '20 (M)
GMAT 1: 760 Q50 V42
GMAT 2: 740 Q49 V42 (Online)
GMAT 3: 760 Q50 V42 (Online)
GPA: 3.8
WE:Marketing (Non-Profit and Government)
Send PM
Re: GMATCLUB VERBAL ATTACK [#permalink]
Expert Reply
Jp27 wrote:
Hi Souvik - Do you have any set that comprises of Number and percentage CR? I guess they appear more often in all types of question. Any such set will be of great help......

Eagerly awaiting for your response....


Here you go
Critical Reasoning Problems: Numbers and Statistics (the Overlap between verbal and quantitative reasoning)
Critical Reasoning: Problems with Numbers and Statistics (part 2)

Also you can check this thursdays-with-ron-critical-reasoning-series-critical-140883.html

Souvik
GMAT Club Bot
Re: GMATCLUB VERBAL ATTACK [#permalink]
 1   2   
Moderators:
GMAT Club Verbal Expert
6919 posts
GMAT Club Verbal Expert
238 posts
GRE Forum Moderator
13958 posts

Powered by phpBB © phpBB Group | Emoji artwork provided by EmojiOne